Download as pdf or txt
Download as pdf or txt
You are on page 1of 38

Stroke NCLEX All

Study online at https://quizlet.com/_5lvk8u


In promoting health maintenance for prevention of strokes, the C: Individuals with hypertension and diabetes- The highest risk
nurse understands that the highest risk for the most common type factors for thrombotic stroke are hypertension and diabetes.
of stroke is present in African Americans have a higher risk for stroke than do white
a. African Americans persons but probably because they have a greater incidence of
b. women who smoke hypertension. Factors such as obesity, diet high in saturated fats
c.individuals with hypertension and diabetes and cholesterol, cigarette smoking, and excessive alcohol use are
d. those who are obese with high dietary fat intake also risk factors but carry less risk than hypertension.
A thrombus that develops in a cerebral artery does not always
C: Circulation through the circle of Willis may provide blood supply
cause a loss of neurologic function because
to the affected area of the brain.
a. the body can dissolve the atherosclerotic plaques as they form
The communication between cerebral arteries in the circle of
b. some tissues of the brain do not require constant blood supply
Willing provides a collateral circulation, which may maintain cir-
to prevent damage
culation to an area of the brain if its original blood supply is
c. circulation through the circle of Willis may provide blood supply
obstructed. ALl areas of the brain require constant blood supply,
to the affected area of the brain
and atherosclerotic plaques are not readily reversed. Neurologic
d. neurologic deficits occur only when major arteries are occluded
deficits can result from ischemia cause by many factors.
by thrombus formation around an atherosclerotic plaque
A patient comes to the emergency department immediately after
experiencing numbness of the face and an inability to speak, but
while the patient awaits examination, the symptoms disappear
and the patient request discharge. The nurse stresses that it is D: The patient has probably experienced a transient ischemic
important for the patient to be evaluated primarily because attack (TIA), which is a sign of progressive cerebral vascular
a. the patient has probably experienced an asymptomatic lacunar disease- A TIA is a temporary focal loss of neurologic function
stroke caused by ischemia of an area of the brain, usually lasting only
b. the symptoms are likely to return and progress to worsening about 3 hours. TIAs may be due to microemboli from heart disease
neurologic deficit in the next 24 hours or carotid or cerebral thrombi and are a warning of progressive
c. neurologic deficits that are transient occur most often as a disease. Evaluation is necessary to determine the cause of the
result of small hemorrhages that clot off neurologic deficit and provide prophylactic treatment if possible.
d. the patient has probably experienced a transient ischemic
attack (TIA), which is a sign of progressive cerebral vascular
disease
The neurologic functions that are affected by a stroke are primarily C: The brain area perfused by the affected artery- clinical mani-
related to festation of altered neurologic function differ, depending primarily
a. the amount of tissue area involved on the specific cerebral artery involved and the area of the brain
b. the rapidity of onset of symptoms that is perfused by the artery. The degree of impairment depends
c. the brain area perfused by the affected artery on rapidity of onset, the size of the lesion, and the presence of
d. the presence or absence of collateral circulation collateral circulation.
A: CT scan- A CT scan is the most commonly used diagnostic test
A patient is admitted to the hospital with a left hemiplegia. To to determine the size and location of the lesion and to differentiate
determine the size and location and to ascertain whether a stroke a thrombotic stroke from a hemorrhagic stroke. Positron emission
is ischemic or hemorrhagic, the nurse anticipates that the health tomography (PET) will show the metabolic activity of the brain
care provider will request a and provide a depiction of the extent of tissue damage after a
a. CT scan stroke. Lumbar punctures are not performed routinely because
b. lumbar puncture of the chance of increased intracranial pressure causing herni-
c. cerebral arteriogram ation. Cerebral arteriograms are invasive and may dislodge an
d. positron emission tomography (PET) embolism or cause further hemorrhage; they are performed only
when no other test can provide the needed information.
A carotid endarterectomy is being considered as a treatment for C: Involves removing an atherosclerotic plaque in the carotid
a patient who has had several TIAs. The nurse explains to the artery to prevent an impending stroke- An endarterectomy is a
patient that this surgery removal of an atherosclerotic plaque, and plaque in the carotid
a. is used to restore blood to the brain following an obstruction of artery may impair circulation enough to cause a stroke. A carotid
a cerebral artery endarterectomy is performed to prevent a cerebrovascular ac-
b. involves intracranial surgery to join a superficial extracranial cident (CVA), as are most other surgical procedures. An ex-
artery to an intracranial artery tacranial-intracranial bypass involves cranial surgery to bypass a
c. involves removing an atherosclerotic plaque in the carotid sclerotic intacranial artery. Percutaneous transluminal angioplasty
artery to prevent an impending stroke uses a balloon to compress stenotic areas in the carotid and
d. is sued to open a stenosis in a carotid artery with a balloon and vertebrobasilar arteries and often includes inserting a stent to hold
stent to restore cerebral circulation the artery open.
C: Daily low dose aspirin- the administration of antiplatelet
agents, such as aspirin, dipyridamole (Persantine), and ticlopdip-
ine (Ticlid), reduces the incidence of stroke in those at risk. An-

1/9
Stroke NCLEX All
Study online at https://quizlet.com/_5lvk8u
ticoagulants are also used for prevention of embolic strokes but
The incidence of ischemic stroke in patients with TIAs and other increase the risk for hemorrhage. Diuretics are not indicated for
risk factors is reduced with administration of stroke prevention other than for their role in controlling BP, and
a. furosemide (Lasix) antilipemic agents have bot been found to have a significant effect
b. lovastatin (Mevacor) on stroke prevention. The calcium channel blocker nimodipine is
c. daily low dose aspirin used in patients with subarachnoid hemorrhage to decrease the
d. nimodipine (Nimotop) effects of vasospasm and minimize tissue damage.
P.S. I freaking love you and good luck on the final!!
The priority intervention in the emergency department for the D: Maintenance of respiratory function with a patent airway and
patient with a stroke is oxygen administration- the first priority in acute management of
a. intravenous fluid replacement the patient with a stroke is preservation of life. Because the patient
b. administration of osmotic diuretics to reduce cerebral edema with a stroke may be unconscious or have a reduced gag reflex,
c. initiation of hypothermia to decrease the oxygen needs of the it is most important to maintain a patent airway for the patient and
brain provide oxygen if respiratory effort is impaired. IV fluid replace-
d. maintenance of respiratory function with a patent airway and ment, treatment with osmotic diuretics, and perhaps hypothermia
oxygen administration may be used for further treatment.
B: Surgical clipping of they aneurysm- Surgical management with
A diagnosis of a ruptured cerebral aneurysm has been made in a clipping of an aneurysm to decrease re bleeding and vasospasm
patient with manifestations of a stroke. The nurse anticipates that is an option for a stroke cause by rupture of a cerebral aneurysm.
treatment options that would be evaluated for the patient include Placement of coils into the lumens of the aneurysm by inter-
a. hyperventilation therapy centional radiologists is increasing in popularity. Hyperventilation
b. surgical clipping of the aneurysm therapy would increase vasodilation and the potential for hemor-
c. administration of hyperosmotic agents rhage. Thrombolytic therapy would be absolutely contraindicated,
d. administration of thrombolytic therapy and if a vessel is patent, osmotic diuretics may leak into tissue,
pulling fluid out of the vessel and increasing edema.
During the acute phase of a stroke, the nurse assesses the
patient's vital signs and neurologic status every 4 hours. A car-
A: Hypertension- The body responds to the vasopasm and a
diovascular sign that the nurse would see as the body attempts
decreased circulation to the brain that occurs with a stroke by
to increase cerebral blood flow is
increasing the BP, frequently resulting in hypertension. The other
a. hypertension
options are important cardiovascular factors to assess, but they
b. fluid overload
do not result from impaired cerebral blood flow.
c. cardiac dysrhythmias
d. S3 and S4 heart sounds
D: Having the patient perform passive ROM of the affected limb
with the unaffected limb- active ROM should be initiated on the
A nursing intervention is indicated for the patient with hemiplegia
unaffected side as soon as possible, and passive ROM of the
is
affected side should be started on the first day. Having the patient
a. the use of a footboard to prevent plantar flexion
actively exercise the unaffected side provides the patient with
b. immobilization of the affected arm against the chest with a sling
active and passive ROM as needed. Use of footboards is contro-
c. positioning the patient in bed with each joint lower than the joint
versial because they stimulate plantar flexion. The unaffected arm
proximal to it
should be supported, but immobilization may precipitate a painful
d. having the patient perform passive ROM of the affected limb
shoulder-hand syndrome. The patient should be positioned with
with the unaffected limb
each joint higher than the joint proximal to it to prevent dependent
edema.
A newly admitted patient who has suffered a right sided brain
A: Place objects on the right side within the patient's field of
stroke has a nursing diagnosis of disturbed visual sensory per-
vision- the presence of homonymous hemianopia in a patient
ception related to homonymous hemianopsia. Early in the care of
with right-hemisphere brain damage causes a loss of vision in
the patient, the nurse should
the left field. Early in the care of the patient, objects should be
a. place objects on the right side within the patient's field of vision
placed on the right side of the patient in the field of vision, and
b. approach the patient from the left side to encourage the patient
the nurse should approach the patient from the right side. Later
to turn the head
in treatment, patients should be taught to turn the head and scan
c. place objects on the patient's left side to assess the patient's
the environment and should be approached from the affected side
ability to compensate
to encourage head turning. Eye patches are used if patients have
d. patch the affected eye to encourage the patient to turn the head
diplopia (double vision).
to scan the environment
Four days following a stroke, a patient is to start oral fluids and A: check the patient's gag reflex- the first step in providing oral
feedings. Before feeding the patient, the nurse should first feedings for a patient with a stroke is ensuring that the patient has
a. check the patient's gag reflex an intact gag reflex because oral feedings will not be provided
b. order a soft diet for the patient if gag reflex is impaired. The nurse should then evaluate the

2/9
Stroke NCLEX All
Study online at https://quizlet.com/_5lvk8u
c. raise the head of the bed to sitting position patient's ability to swallow ice chips or ice water after placing the
d. evaluate the patient's ability to swallow small sips of ice water patient in an upright position
C: scrambled eggs- soft foods that provide enough texture, flavor,
An appropriate food for a patient with a stroke who has mild
and bulk to stimulate swallowing should be used for the patient
dysphagia is
with dysphasia. Thin liquids are difficult to swallow, and patients
a. fruit juices
may not be able to control them in the mouth. Pureed foods
b. pureed meat
are often too bland and to smooth, and milk products should be
c. scrambled eggs
avoided because they tend to increase the viscosity of mucus and
d. fortified milkshakes
increase salivation.
A patient's wife asks the nurse why her husband did not receive
the clot busting medication (tPA) she has been reading about. Her D: The medication you are talking about dissolves clots and could
husband is diagnosed with a hemorrhagic stroke. What should the cause more bleeding in your husband's head- tPA dissolves clots
nurse respond? and increases the risk for bleeding. It is not used with hemorrhagic
a. He didn't arrive within the time frame for that therapy strokes. If the patient had a thrombotic/embolic stroke the time
b. Not every is eligible for this drug. Has he had surgery lately? frame would be important as well as a history of surgery. The nurse
c. You should discuss the treatment of your husband with your should answer the question as accurately as possible and then
doctor encourage the individual to talk with the primary care physician if
d. The medication you are talking about dissolves clots and could he or she has further questions.
cause more bleeding in your husband's head
To promote communication during rehabilitation of the patient with
aphasia, an appropriate nursing intervention is to B: Talk about ADLs that are familiar to the patient- during rehabili-
a. use gestures, pictures, and music to stimulate patient respons- tation, the patient with aphasia needs frequent, meaningful verbal
es stimulation that has relevance for him. Conversation by the nurse
b. talk about activities of daily living (ADLs) that are familiar to the and family should address ADLs that are familiar to the patient.
patient Gestures, pictures, and simple statements are more appropriate
c. structure statements so that patient does not have to respond in the acute phase, when patients may be overwhelmed with
verbally verbal stimuli. Flashcards are often perceived by the patient as
d. use flashcards with simple words and pictures to promote childish and meaningless.
language recall
C: Teach the patient to care consciously for the affected side-
A patient with right hemisphere stroke has a nursing diagnosis unilateral neglect, or neglect syndrome, occurs when the patient
of unilateral neglect related to sensory perceptual deficits. During with a stroke is unaware of the affected side of the body, which
the patient's rehabilitation, it is important for the nurse to puts the patient at risk for injury. During the acute phase, the
a. avoid positioning the patient on the affected side affected side is cared for by the nurse with positioning and sup-
b. place all objects for care on the patient's unaffected side port, during rehabilitation the patient is taught to care consciously
c. teach the patient to care consciously for the affected side for and attend to the affected side of the body to protect it from
d. protect the affected side from injury with pillows and supports injury. Patients may be positioned on the affected side for up to 30
minutes.
C: Distract the patient from inappropriate emotional responses-
A patient with a stroke has a right sided hemiplegia. The nurse
patients with left-sided brain damage from stroke often experi-
prepares family members to help control behavior changes seen
ence emotional lability, inappropriate emotional responses, mood
with this type of stroke by teaching them to
swings, and uncontrolled tears or laughter disproportionate or out
a. ignore undesirable behaviors manifested by the patient
of context with the situation. The behavior is upsetting and embar-
b. provide directions to the patient verbally in small steps
rassing to both the patient and the family, and the patient should
c. distract the patient from inappropriate emotional responses
be distracted to minimize its presence. Patients with right-brain
d. supervise all activities before allowing the patient to pursue
damage often have impulsive, rapid behavior that supervision and
them independently
direction.
The nurse can assist the patient and the family in coping with the D: Helping the patient and family understand the significance of
long term effects of a stroke by residual stroke damage to promote problem solving and planning-
a. informing family members that the patient will need assistance the patient and family need accurate and complete information
with almost all ADLs about the effects of the stroke to problem solve and make plans for
b. explaining that the patient's prestroke behavior will return as chronic care of the patient. It is uncommon for patients with major
improvement progresses strokes to return completely to pre stroke function, behaviors, and
c. encouraging the patient and family members to seek assis- role, and both the patient and family will mourn these losses. The
tance from family therapy or stroke support groups patient's specific needs for care must be identified, and rehabil-
d. helping the patient and family understand the significance of itation efforts should be continued at home. Family therapy and
residual stroke damage to promote problem solving and planning support groups may be helpful for some patients and families.
Which intervention should the nurse delegate to the LPN when
caring for a patient following an acute stroke?
a. assess the patient's neurologic status
3/9
Stroke NCLEX All
Study online at https://quizlet.com/_5lvk8u
b. assess the patient's gag reflex before beginning feeding C: Administer ordered antihypertensives and platelet inhibitors-
c. administer ordered antihypertensives and platelet inhibitors medication administration is within the scope of practice for an
d. teach the patient's caregivers strategies to minimize unilateral LPN. Assessment and teaching are within the scope of practice
neglect for the RN.
Type most often signaled by TIAs, commonly occurs during or
Characteristics of a thrombotic stroke
after sleep, strong association with hypertension
Onset unrelated to activity, quick onset and resolution, associated
Characteristics of embolic stroke
with endocardial disorders
Rupture of atherosclerotic vessels, carries the poorest prognosis,
Characteristics of intracerebral hemorrhage
creates mass that compresses the brain
High initial mortality, symptoms of meningeal irritation, caused by
Characteristics of subarachnoid hemorrhage rupture of intracranial aneurysm, associated with sudden, severe
headache
Manifestations of left brain damage Aphasia, inability to remember words
Left homonymous hemianopsia, agnosia, quick impulsive behav-
Manifestations of right brain damage
ior, neglect of the left side of the body
Which of the following is the best treatment for acute ischemic
stroke?
a. heparin
b. LMWH
c. Alteplase
d. Eptifibatie
e. Warfarin
C: Alteplase
Which of the following is the best treatment for acute ischemic
stroke?
a. heparin
b. LMWH
c. Alteplase
d. Eptifibatie
e. Warfarin
1st thing you do when you suspect a stroke Non-contrast CT scan (determines hemorrhagic vs ischemic)
Most common artery involved Internal Carotid Artery
If you drop blood pressure then more green tissue (ischemic) turns
Permissive hypertension red (infarcted)
(only treat BP if >200)
Treat for hemorrhagic stroke Lower BP
Problem with IV-TPA only 1-3% of patients arrive in time to receive it
1. Intracranial vascular disease
Arterial sources of stroke (3) 2. Carotid vascular disease
3. Aortic arch
1. Anticoagulants
Secondary prevention of stroke (3) 2. Antiplatelets
3. Surgery
1. smoking cessation
2. diabetes control
Secondary prevention: Risk factor modification (4)
3. aggressive cholesterol lowering
4. hypertension control
1)deep hypertensive intracerebral hemorrhages, 2)ruptured sac-
4 major causes of hemorrhagic stroke cular aneurysms, 3)arteriovenous malformation,4)spontaneous
lobar hemorrhages

After a patient experienced a brief episode of tinnitus, diplopia,


C. The patient's symptoms are consistent with transient ischemic
and dysarthria with no residual effects, the nurse anticipates
attack (TIA), and drugs that inhibit platelet aggregation are pre-
teaching the patient about
scribed after a TIA to prevent stroke. Continuous heparin infusion
a. cerebral aneurysm clipping.
is not routinely used after TIA or with acute ischemic stroke. The
b. heparin intravenous infusion.
4/9
Stroke NCLEX All
Study online at https://quizlet.com/_5lvk8u
c. oral low-dose aspirin therapy. patient's symptoms are not consistent with a cerebral aneurysm.
d. tissue plasminogen activator (tPA). tPA is used only for acute ischemic stroke, not for TIA.
A 68-year-old patient is being admitted with a possible stroke.
Which information from the assessment indicates that the nurse
should consult with the health care provider before giving the
prescribed aspirin? C. A sudden onset headache is typical of a subarachnoid hemor-
a. The patient has dysphasia. rhage, and aspirin is contraindicated. Atrial fibrillation, dysphasia,
b. The patient has atrial fibrillation. and transient ischemic attack (TIA) are not contraindications to
c. The patient reports that symptoms began with a severe aspirin use, so the nurse can administer the aspirin.
headache.
d. The patient has a history of brief episodes of right-sided hemi-
plegia.
A 73-year-old patient with a stroke experiences facial drooping
on the right side and right-sided arm and leg paralysis. When
admitting the patient, which clinical manifestation will the nurse D. Right-sided paralysis indicates a left-brain stroke, which will
expect to find? lead to difficulty with comprehension and use of language. The
a. Impulsive behavior left-side reflexes are likely to be intact. Impulsive behavior and
b. Right-sided neglect neglect are more likely with a right-side stroke.
c. Hyperactive left-sided tendon reflexes
d. Difficulty comprehending instructions
During the change of shift report a nurse is told that a patient has
an occluded left posterior cerebral artery. The nurse will anticipate
C. Visual disturbances are expected with posterior cerebral artery
that the patient may have
occlusion. Aphasia occurs with middle cerebral artery involve-
a. dysphasia.
ment. Cognitive deficits and changes in judgment are more typical
b. confusion.
of anterior cerebral artery occlusion.
c. visual deficits.
d. poor judgment.
When teaching about clopidogrel (Plavix), the nurse will tell the
D. Clopidogrel (Plavix) inhibits platelet function and increases the
patient with cerebral atherosclerosis
risk for gastrointestinal bleeding, so patients should be advised
a. to monitor and record the blood pressure daily.
to notify the health care provider about any signs of bleeding.
b. that Plavix will dissolve clots in the cerebral arteries.
The medication does not lower blood pressure, decrease plaque
c. that Plavix will reduce cerebral artery plaque formation.
formation, or dissolve clots.
d. to call the health care provider if stools are bloody or tarry.
A patient with carotid atherosclerosis asks the nurse to describe
a carotid endarterectomy. Which response by the nurse is accu-
A. In a carotid endarterectomy, the carotid artery is incised and
rate?
the plaque is removed. The response beginning, "The diseased
a. "The obstructing plaque is surgically removed from an artery in
portion of the artery in the brain is replaced" describes an arterial
the neck."
graft procedure. The answer beginning, "A catheter with a deflated
b. "The diseased portion of the artery in the brain is replaced with
balloon is positioned at the narrow area" describes an angioplasty.
a synthetic graft."
The final response beginning, "A wire is threaded through the
c. "A wire is threaded through an artery in the leg to the clots in
artery" describes the mechanical embolus removal in cerebral
the carotid artery and the clots are removed."
ischemia (MERCI) procedure.
d. "A catheter with a deflated balloon is positioned at the narrow
area, and the balloon is inflated to flatten the plaque."
A patient admitted with possible stroke has been aphasic for 3
D. Because elevated BP may be a protective response to main-
hours and his current blood pressure (BP) is 174/94 mm Hg.
tain cerebral perfusion, antihypertensive therapy is recommended
Which order by the health care provider should the nurse ques-
only if mean arterial pressure (MAP) is >130 mm Hg or systolic
tion?
pressure is >220 mm Hg. Fluid intake should be 1500 to 2000
a. Keep head of bed elevated at least 30 degrees.
mL daily to maintain cerebral blood flow. The head of the bed
b. Infuse normal saline intravenously at 75 mL/hr.
should be elevated to at least 30 degrees, unless the patient has
c. Administer tissue plasminogen activator (tPA) per protocol.
symptoms of poor tissue perfusion. tPA may be administered if the
d. Administer a labetalol (Normodyne) drip to keep BP less than
patient meets the other criteria for tPA use.
140/90 mm Hg.
A 56-year-old patient arrives in the emergency department with
D. The patient's history and clinical manifestations suggest an
hemiparesis and dysarthria that started 2 hours previously, and
acute ischemic stroke and a patient who is seen within 4.5 hours
health records show a history of several transient ischemic attacks
of stroke onset is likely to receive tPA (after screening with a
(TIAs). The nurse anticipates preparing the patient for
CT scan). Heparin administration in the emergency phase is
a. surgical endarterectomy.
not indicated. Emergent carotid transluminal angioplasty or en-
b. transluminal angioplasty.

5/9
Stroke NCLEX All
Study online at https://quizlet.com/_5lvk8u
c. intravenous heparin administration. darterectomy is not indicated for the patient who is having an
d. tissue plasminogen activator (tPA) infusion. acute ischemic stroke.
A female patient who had a stroke 24 hours ago has expressive
aphasia. The nurse identifies the nursing diagnosis of impaired
A. Communication will be facilitated and less frustrating to the
verbal communication. An appropriate nursing intervention to help
patient when questions that require a "yes" or "no" response
the patient communicate is to
are used. When the language areas of the brain are injured,
a. ask questions that the patient can answer with "yes" or "no."
the patient might not be able to read or recite words, which will
b. develop a list of words that the patient can read and practice
frustrate the patient without improving communication. Expressive
reciting.
aphasia is caused by damage to the language areas of the brain,
c. have the patient practice her facial and tongue exercises with a
not by the areas that control the motor aspects of speech. The
mirror.
nurse should allow time for the patient to respond.
d. prevent embarrassing the patient by answering for her if she
does not respond.
A 72-year-old patient who has a history of a transient ischemic
attack (TIA) has an order for aspirin 160 mg daily. When the nurse
is administering medications, the patient says, "I don't need the C. Aspirin is ordered to prevent stroke in patients who have ex-
aspirin today. I don't have a fever." Which action should the nurse perienced TIAs. Documentation of the patient's refusal to take the
take? medication is an inadequate response by the nurse. There is no
a. Document that the aspirin was refused by the patient. need to clarify the order with the health care provider. The aspirin
b. Tell the patient that the aspirin is used to prevent a fever. is not ordered to prevent aches and pains.
c. Explain that the aspirin is ordered to decrease stroke risk.
d. Call the health care provider to clarify the medication order.
For a patient who had a right hemisphere stroke the nurse estab-
A. The patient with right-sided brain damage typically denies any
lishes a nursing diagnosis of
deficits and has poor impulse control, leading to risk for injury
a. risk for injury related to denial of deficits and impulsiveness.
when the patient attempts activities such as transferring from a
b. impaired physical mobility related to right-sided hemiplegia.
bed to a chair. Right-sided brain damage causes left hemiple-
c. impaired verbal communication related to speech-language
gia. Left-sided brain damage typically causes language deficits.
deficits.
Left-sided brain damage is associated with depression and dis-
d. ineffective coping related to depression and distress about
tress about the disability.
disability.
A patient in the clinic reports a recent episode of dysphasia and
B. Following a transient ischemic attack (TIA), patients typically
left-sided weakness at home that resolved after 2 hours. The
are started on medications such as aspirin to inhibit platelet
nurse will anticipate teaching the patient about
function and decrease stroke risk. tPA is used for acute ischemic
a. alteplase (tPA).
stroke. Coumadin is usually used for patients with atrial fibrillation.
b. aspirin (Ecotrin).
Nimodipine is used to prevent cerebral vasospasm after a sub-
c. warfarin (Coumadin).
arachnoid hemorrhage.
d. nimodipine (Nimotop).
When caring for a patient with a new right-sided homonymous
C. During the acute period, the nurse should place objects on the
hemianopsia resulting from a stroke, which intervention should
patient's unaffected side. Because there is a visual defect in the
the nurse include in the plan of care?
right half of each eye, an eye patch is not appropriate. The patient
a. Apply an eye patch to the right eye.
should be approached from the left side. The visual deficit may
b. Approach the patient from the right side.
not resolve, although the patient can learn to compensate for the
c. Place objects needed on the patient's left side.
defect.
d. Teach the patient that the left visual deficit will resolve.
A 58-year-old patient with a left-brain stroke suddenly bursts into
D. Patients who have left-sided brain stroke are prone to emotional
tears when family members visit. The nurse should
outbursts that are not necessarily related to the emotional state
a. use a calm voice to ask the patient to stop the crying behavior.
of the patient. Depression after a stroke is common, but the
b. explain to the family that depression is normal following a stroke.
suddenness of the patient's outburst suggests that depression is
c. have the family members leave the patient alone for a few
not the major cause of the behavior. The family should stay with
minutes.
the patient. The crying is not within the patient's control and asking
d. teach the family that emotional outbursts are common after
the patient to stop will lead to embarrassment.
strokes.
The nurse identifies the nursing diagnosis of imbalanced nutri-
tion: less than body requirements related to impaired self-feeding C. Because the nursing diagnosis indicates that the patient's
ability for a left-handed patient with left-sided hemiplegia. Which imbalanced nutrition is related to the left-sided hemiplegia, the
intervention should be included in the plan of care? appropriate interventions will focus on teaching the patient to
a. Provide a wide variety of food choices. use the right hand for self-feeding. The other interventions are
b. Provide oral care before and after meals. appropriate for patients with other etiologies for the imbalanced
c. Assist the patient to eat with the right hand. nutrition.
d. Teach the patient the "chin-tuck" technique.

6/9
Stroke NCLEX All
Study online at https://quizlet.com/_5lvk8u
Which stroke risk factor for a 48-year-old male patient in the clinic
C. Hypertension is the single most important modifiable risk factor.
is most important for the nurse to address?
People who drink more than 1 (for women) or 2 (for men) alcoholic
a. The patient is 25 pounds above the ideal weight.
beverages a day may increase risk for hypertension. Physical
b. The patient drinks a glass of red wine with dinner daily.
inactivity and obesity contribute to stroke risk but not as much as
c. The patient's usual blood pressure (BP) is 170/94 mm Hg.
hypertension.
d. The patient works at a desk and relaxes by watching television.
A 40-year-old patient has a ruptured cerebral aneurysm and A. The patient with a subarachnoid hemorrhage usually has mini-
subarachnoid hemorrhage. Which intervention will be included in mal activity to prevent cerebral vasospasm or further bleeding and
the care plan? is at risk for venous thromboembolism (VTE). Activities such as
a. Apply intermittent pneumatic compression stockings. coughing and sitting up that might increase intracranial pressure
b. Assist to dangle on edge of bed and assess for dizziness. (ICP) or decrease cerebral blood flow are avoided. Because there
c. Encourage patient to cough and deep breathe every 4 hours. is no indication that the patient is unconscious, an oropharyngeal
d. Insert an oropharyngeal airway to prevent airway obstruction. airway is inappropriate.
A patient in the emergency department with sudden-onset D. The use of warfarin probably contributed to the intracerebral
right-sided weakness is diagnosed with an intracerebral hemor- bleeding and remains a risk factor for further bleeding. Admin-
rhage. Which information about the patient is most important to istration of vitamin K is needed to reverse the effects of the
communicate to the health care provider? warfarin, especially if the patient is to have surgery to correct
a. The patient's speech is difficult to understand. the bleeding. The history of hypertension is a risk factor for the
b. The patient's blood pressure is 144/90 mm Hg. patient but has no immediate effect on the patient's care. The BP
c. The patient takes a diuretic because of a history of hyperten- of 144/90 indicates the need for ongoing monitoring but not for any
sion. immediate change in therapy. Slurred speech is consistent with a
d. The patient has atrial fibrillation and takes warfarin (Coumadin). left-sided stroke, and no change in therapy is indicated.
A 47-year-old patient will attempt oral feedings for the first time
C. The patient should be as upright as possible before attempting
since having a stroke. The nurse should assess the gag reflex and
feeding to make swallowing easier and decrease aspiration risk.
then
To assess swallowing ability, the nurse should initially offer water
a. order a varied pureed diet.
or ice to the patient. Pureed diets are not recommended because
b. assess the patient's appetite.
the texture is too smooth. The patient may have a poor appetite,
c. assist the patient into a chair.
but the oral feeding should be attempted regardless.
d. offer the patient a sip of juice.
A patient with left-sided weakness that started 60 minutes earlier D. Rapid screening with a noncontrast CT scan is needed before
is admitted to the emergency department and diagnostic tests are administration of tissue plasminogen activator (tPA), which must
ordered. Which test should be done first? be given within 4.5 hours of the onset of clinical manifestations of
a. Complete blood count (CBC) the stroke. The sooner the tPA is given, the less brain injury. The
b. Chest radiograph (Chest x-ray) other diagnostic tests give information about possible causes of
c. 12-Lead electrocardiogram (ECG) the stroke and do not need to be completed as urgently as the CT
d. Noncontrast computed tomography (CT) scan scan.
A male patient who has right-sided weakness after a stroke is
making progress in learning to use the left hand for feeding and
other activities. The nurse observes that when the patient's wife
is visiting, she feeds and dresses him. Which nursing diagnosis is C. The information supports the diagnosis of disabled family
most appropriate for the patient? coping because the wife does not understand the rehabilitation
a. Interrupted family processes related to effects of illness of a program. There are no data supporting low self-esteem, and the
family member patient is attempting independence. The data do not support an
b. Situational low self-esteem related to increasing dependence interruption in family processes because this may be a typical
on spouse for care pattern for the couple. There is no indication that the patient has
c. Disabled family coping related to inadequate understanding by impaired nutrition.
patient's spouse
d. Impaired nutrition: less than body requirements related to hemi-
plegia and aphasia
Nurses in change-of-shift report are discussing the care of a
patient with a stroke who has progressively increasing weakness
and decreasing level of consciousness (LOC). Which nursing di- D. Protection of the airway is the priority of nursing care for a
agnosis do they determine has the highest priority for the patient? patient having an acute stroke. The other diagnoses are also
a. Impaired physical mobility related to weakness appropriate, but interventions to prevent aspiration are the priority
b. Disturbed sensory perception related to brain injury at this time.
c. Risk for impaired skin integrity related to immobility
d. Risk for aspiration related to inability to protect airway
Several weeks after a stroke, a 50-year-old male patient has
impaired awareness of bladder fullness, resulting in urinary in-
continence. Which nursing intervention will be best to include in
7/9
Stroke NCLEX All
Study online at https://quizlet.com/_5lvk8u
the initial plan for an effective bladder training program? B. Developing a regular voiding schedule will prevent inconti-
a. Limit fluid intake to 1200 mL daily to reduce urine volume. nence and may increase patient awareness of a full bladder. A
b. Assist the patient onto the bedside commode every 2 hours. 1200 mL fluid restriction may lead to dehydration. Intermittent
c. Perform intermittent catheterization after each voiding to check catheterization and use of a condom catheter are appropriate in
for residual urine. the acute phase of stroke, but should not be considered solutions
d. Use an external "condom" catheter to protect the skin and for long-term management because of the risks for urinary tract
prevent embarrassment. infection (UTI) and skin breakdown.
Which information about the patient who has had a subarachnoid B. To prevent cerebral vasospasm and maintain cerebral perfu-
hemorrhage is most important to communicate to the health care sion, blood pressure needs to be maintained at a level higher
provider? than 90 mm Hg systolic after a subarachnoid hemorrhage. A low
a. The patient complains of having a stiff neck. BP or drop in BP indicates a need to administer fluids and/or
b. The patient's blood pressure (BP) is 90/50 mm Hg. vasopressors to increase the BP. An ongoing headache, RBCs
c. The patient reports a severe and unrelenting headache. in the CSF, and a stiff neck are all typical clinical manifestations
d. The cerebrospinal fluid (CSF) report shows red blood cells of a subarachnoid hemorrhage and do not need to be rapidly
(RBCs). communicated to the health care provider.
The nurse is caring for a patient who has been experiencing stroke
symptoms for 60 minutes. Which action can the nurse delegate to
C. Administration of subcutaneous medications is included in
a licensed practical/vocational nurse (LPN/LVN)?
LPN/LVN education and scope of practice. The other actions
a. Assess the patient's gag and cough reflexes.
require more education and scope of practice and should be done
b. Determine when the stroke symptoms began.
by the registered nurse (RN).
c. Administer the prescribed short-acting insulin.
d. Infuse the prescribed IV metoprolol (Lopressor).
After receiving change-of-shift report on the following four pa-
tients, which patient should the nurse see first?
a. A 60-year-old patient with right-sided weakness who has an
infusion of tPA prescribed A. tPA needs to be infused within the first few hours after stroke
b. A 50-year-old patient who has atrial fibrillation and a new order symptoms start in order to be effective in minimizing brain injury.
for warfarin (Coumadin) The other medications should also be given as quickly as possible,
c. A 40-year-old patient who experienced a transient ischemic but timing of the medications is not as critical.
attack yesterday who has a dose of aspirin due
d. A 30-year-old patient with a subarachnoid hemorrhage 2 days
ago who has nimodipine (Nimotop) scheduled
The nurse is caring for a patient who has just returned after having B. Small emboli can occur during carotid artery angioplasty and
left carotid artery angioplasty and stenting. Which assessment stenting, and the aphasia indicates a possible stroke during the
information is of most concern to the nurse? procedure. Slightly elevated pulse rate and blood pressure are
a. The pulse rate is 102 beats/min. not unusual because of anxiety associated with the procedure.
b. The patient has difficulty speaking. Fine crackles at the lung bases may indicate atelectasis caused
c. The blood pressure is 144/86 mm Hg. by immobility during the procedure. The nurse should have the
d. There are fine crackles at the lung bases. patient take some deep breaths.
A 70-year-old female patient with left-sided hemiparesis arrives
by ambulance to the emergency department. Which action should
C. The initial nursing action should be to assess the airway and
the nurse take first?
take any needed actions to ensure a patent airway. The other ac-
a. Monitor the blood pressure.
tivities should take place quickly after the ABCs (airway, breathing,
b. Send the patient for a computed tomography (CT) scan.
and circulation) are completed.
c. Check the respiratory rate and effort.
d. Assess the Glasgow Coma Scale score.
The home health nurse is caring for an 81-year-old who had a
B. The spouse's household and patient care responsibilities, in
stroke 2 months ago. Based on information shown in the ac-
combination with chronic illnesses, indicate a high risk for care-
companying figure from the history, physical assessment, and
giver role strain. The nurse should further assess the situation
physical therapy/occupational therapy, which nursing diagnosis is
and take appropriate actions. The data about the control of the
the highest priority for this patient?
patient's diabetes indicates that ineffective health maintenance
a. Impaired transfer ability
and risk for unstable blood glucose are not priority concerns at
b. Risk for caregiver role strain
this time. Because the patient is able to ambulate with a cane, the
c. Ineffective health maintenance
nursing diagnosis of impaired transfer ability is not supported.
d. Risk for unstable blood glucose level
A 63-year-old patient who began experiencing right arm and leg
weakness is admitted to the emergency department. In which C, D, A, B
order will the nurse implement these actions included in the stroke The initial actions should be those that help with airway, breathing,
protocol? and circulation. Baseline neurologic assessments should be done
a. Obtain computed tomography (CT) scan without contrast.
8/9
Stroke NCLEX All
Study online at https://quizlet.com/_5lvk8u
b. Infuse tissue plasminogen activator (tPA).
next. A CT scan will be needed to rule out hemorrhagic stroke
c. Administer oxygen to keep O2 saturation >95%.
before tPA can be administered.
d. Use National Institute of Health Stroke Scale to assess patient.

9/9
Stroke NCLEX Practice Questions
Study online at https://quizlet.com/_9l9w70
The nurse caring for a client with a history of transient ischemic
attacks (TIAs) is reviewing medications ordered to prevent a
Beta Blocker
stroke. Which medication therapy requires follow-up?
Even though beta blockers are useful in lowering blood pressure,
Thiazide diuretic
they are very limited in preventing stroke. Anticoagulants and
antiplatelets are used to reduce the risk of stroke in clients with
Anticoagulant
TIAs. Hypertension is the leading cause of stroke. Research in-
dicates that thiazide diuretics and certain other antihypertensives
Antiplatelet
are useful in reducing stroke risk.
Beta blocker
A blood clot lodges in a cerebral vessel and blocks blood flow.
Which description of an acute embolic stroke given by the nurse
is most accurate?
In embolic stroke, a blood clot or other matter traveling through
cerebral blood vessels becomes lodged in a narrow vessel block-
The local cerebral tissue becomes engorged with blood from a
ing blood flow. The area of the brain supplied by the blocked
ruptured cerebral vessel.
vessel becomes ischemic. The clot may originate from a thrombus
formed in the left side of the heart during atrial fibrillation, bacte-
A blood clot lodges in a cerebral vessel and blocks blood flow.
rial endocarditis, recent myocardial infarction (MI), atherosclerotic
plaque from the carotid artery, rheumatic heart disease, or ven-
Infarcted areas in the brain slough off, leaving cavities in the brain
tricular aneurysm. Infarcted areas of the brain become ischemic
tissue.
but do not slough off. Hemorrhagic stroke is when local cerebral
tissue becomes engorged with blood from a ruptured cerebral
Cerebral vascular pressure exceeds the elasticity of the vessel
vessel. An embolic stroke is not the result of cerebral vascular
wall, resulting in hemorrhages.
pressure increases.
Ruptured aneurysm in the brain

Rupture of a fragile arterial vessel in the brain

Traumatic injury to the brain


The nurse is teaching a class about the causes of a hemorrhagic
Arterial bleeds in the brain cause hemorrhagic stroke. Blood en-
stroke. Which should the nurse include? (Select all that apply.)
ters the brain and puts pressure on brain tissue. Manifestations
occur suddenly because of the rapid rise in intracranial pressure
Damage to the blood-brain barrier
(ICP). Aneurysms in the brain enlarge over time. This causes the
arterial walls to become thin and subject to rupturing. Falls and
Ruptured aneurysm in the brain
other traumatic injuries can cause the arterial walls to rupture.
This causes intracranial bleeding with accompanying increased
Atherosclerotic plaque breaking off in the artery
ICP. Stroke caused by traumatic injury has the poorest outcome
with greater likelihood of death. Atherosclerotic plaque that breaks
Rupture of a fragile arterial vessel in the brain
off causes obstruction in the vessel lumen. This is ischemic stroke,
rather than hemorrhagic. Hemorrhagic stroke involves bleeding
Traumatic injury to the brain
into the brain. The
blood-brain barrier prevents potentially harmful substances from
entering the brain. Hemorrhagic stroke is not caused by damage to
the blood-brain barrier. However, hemorrhagic stroke could cause
damage to the blood-brain
barrier and therefore allow harmful substances to enter the brain.
The nurse is teaching a client about the cause of a transient
ischemic attack (TIA). Which should the nurse include?
Brief period of a neurologic deficit
Brief period of a neurologic deficit
A TIA is a type of ischemic stroke resulting from a localized
neurologic deficit lasting 24 hours or less. Vascular blockage is the
Vascular blockage
cause of an embolic stroke. Intracranial bleeds cause hemorrhagic
strokes. A thrombotic stroke is the result of the formation of a clot
Sudden intracranial bleed
in a blood vessel.
Formation of a clot in a blood vessel
A client was diagnosed with a thrombotic stroke of the vertebral
artery. Which assessment does the nurse expect to make?

1/5
Stroke NCLEX Practice Questions
Study online at https://quizlet.com/_9l9w70
Stupor
Dysphagia
Global aphasia
Dysphagia is the clinical manifestation that is associated with a
stroke that affects the vertebral artery. The other clinical manifes-
Contralateral paralysis
tations are seen with internal carotid and middle cerebral artery
involvement.
Dysphagia
An adult client had a stroke involving the internal carotid artery of
the dominant hemisphere. The nurse should anticipate that the
Speaking
client will have difficulty with which function?
Clinical manifestations of a stroke involving the internal carotid
Speaking
artery include contralateral paralysis of face and limbs, contralat-
eral sensory deficits of face and limbs, aphasia, apraxia, agnosia,
Staying alert
unilateral neglect, and homonymous hemianopia. Difficulty swal-
lowing, drowsiness, and urine retention are not expected in this
Retaining urine
type of stroke.
Swallowing
The nurse is caring for a client recovering from a stroke in the
rehabilitation setting. Which is the goal of care during this stage? Improving muscle strength and coordination

Minimizing brain injury During the rehabilitation treatment stage of stroke, the focus is on
client safety and improvement of muscle strength and coordina-
Dispatching rapid emergency medical services (EMS) tion. Priorities during the treatment stage of acute care immedi-
ately following a stroke include rapid EMS dispatch, diagnosing
Diagnosing the type and cause of stroke the type and cause of stroke, and other interventions to minimize
brain injury and maximize client recovery.
Improving muscle strength and coordination
The nurse on the stroke rehabilitation unit is planning care for a
client who is experiencing vision and equilibrium deficits, altered
proprioception, hemianopia, and neglect syndrome. Which nurs-
ing therapy is the most important to include?
Providing reassurance and support
Maintaining fluid, oxygen, and nutritional status
The client with sensory-perceptual deficits needs reassurance
and support. There is no indication that the client cannot maintain
Providing reassurance and support
fluid, oxygen, and nutritional status, cannot communicate well, or
has cognitive or behavioral changes.
Developing an alternate means of communicating

Providing behavioral and cognitive therapy when the condition


stabilizes
The nurse taught a group of clients recovering from a stroke how
to perform active range-of-motion exercises. Which client requires
further teaching?

The client performing flexion, extension, and hyperextension of The client with right-sided paralysis flexing and extending only the
the hips bilaterally left knee

The client with right-sided paralysis flexing and extending only the The client can use the left side to help flex and extend the right
left knee knee. Both sides should be exercised. All the other range-of-mo-
tion exercises are appropriate.
The client performing extension and hyperextension of the neck

The client with left-sided paralysis using the right arm to help flex
and extend the left wrist
The nurse is reviewing interventions aimed at maintaining cere-
bral perfusion in a client who had a thrombotic stroke. Which Encouraging active range-of-motion exercises
intervention should the nurse question?
Active range-of-motion exercises promote physical mobility but
Encouraging active range-of-motion exercises will not directly assist in maintaining cerebral perfusion. The initial

2/5
Stroke NCLEX Practice Questions
Study online at https://quizlet.com/_9l9w70
Placing the client in a side-lying position
focus of care is to identify changes in airway, breathing, and circu-
lation that could indicate decreased cerebral perfusion. Maintain-
Monitoring mental status and level of consciousness
ing adequate oxygenation and positioning to facilitate breathing is
appropriate.
Monitoring respiratory status
A client has a history of transient ischemic attacks (TIAs). Which Antiplatelet
medication does the nurse expect to find in the client's list of
prescriptions? An antiplatelet may be prescribed for clients who have TIAs or who
have had previous strokes. Its purpose is to prevent clot formation
Beta blocker with the resulting vessel occlusion. An oral anticoagulant med-
ication may be prescribed shortly after a stroke to prevent blood
Antiplatelet clot formation and to enhance cerebral blood flow by keeping the
blood thin. A beta blocker is useful for lowering blood pressure but
Anticoagulant is limited in preventing stroke. Docusate sodium (Colace) is a stool
softener that may be prescribed after a stroke to prevent straining
Stool softener at stool, which increases intracranial pressure (ICP).
A client who is diagnosed with a stroke has an order for a tissue
plasminogen activator (tPA). Which circumstance does the nurse
The stroke must have occurred within 3 hours of administering the
suspect is present?
medication.
The stroke must be hemorrhagic in nature.
For the safe administration of tPA, the medication must be admin-
istered within 3 hours of the onset of the symptoms of stroke. The
Aspirin therapy must have been received for 6 months for tPA to
stroke cannot be hemorrhagic in nature because the action of the
be effective.
medication is to dissolve the clot, which would not be intended
for a reclotted ruptured hemorrhagic vessel. There is no minimal
The stroke must have occurred within 3 hours of administering the
or maximal degree of plaque buildup that is necessary for the
medication.
safe administration of the medication. Aspirin therapy is not a
requirement for tPA to be administered.
Atherosclerotic buildup in affected arteries must be greater than
90%.
A client who is diagnosed with stroke is very drowsy but can
respond when awakened. Using the National Institutes of Health 1
Stroke Scale, which level of consciousness should the nurse
document? A score of 1 means that the client is not alert but is arousable
by minor stimulation to obey, answer, or respond. A score of 0
1 means that the client is alert and keenly responsive. A score of
2 means that the client is not alert, requires repeated stimulation
2 to attend, or is obtunded and requires strong or painful stimuli to
make movements. A score of 3 means that the client responds
0 only with motor or autonomic effects or is totally unresponsive,
flaccid, and areflexic.
3
A client who had a stroke secondary to cerebral stenosis dis-
cussed surgical options with the surgeon. Which option should
Carotid angioplasty with stenting
the nurse anticipate will be performed?
Carotid angioplasty with stenting is used to surgically treat cere-
Cautious observation only
bral stenosis. Carotid endarterectomy is used to remove plaque
from a carotid artery. An extracranial-intracranial bypass may be
Extracranial-intracranial bypass
required if an occluded or stenotic vessel is not directly accessible.
The client has already had a stroke from the stenosis, and there
Carotid angioplasty with stenting
is no indication that comorbidities could prevent the surgery.
Carotid endarterectomy
A client diagnosed with a stroke is having difficulty walking and
may require the use of a walker. Which area should the nurse Physical therapy
make a referral to?
Occupational therapy can help a client learn to use assistive de-
Speech and language therapy vices and create a plan for regaining motor skills. Physical therapy
helps increase physical strength and coordination and prevent
Occupational therapy contractures. Speech and language therapy improve communica-

3/5
Stroke NCLEX Practice Questions
Study online at https://quizlet.com/_9l9w70
Home health
tion and swallowing. Home health may be needed, but the priority
is learning to use the assistive device.
Physical therapy
The nurse is reviewing the plan of care for a client who is unre-
sponsive following a stroke. Which intervention should the nurse
question?
Encouraging active range-of-motion exercises
Encouraging active range-of-motion exercises
Each of the nursing implementations listed are appropriate for
promoting physical mobility. However, the client is unresponsive
Elevating the head of the bed 30 degrees
and therefore cannot complete active range-of-motion exercises;
they would require passive range-of-motion exercises.
Turning the client every 2 hours

Monitoring lower extremities for symptoms of thrombophlebitis


The nurse is reviewing documentation of a physical examination
of a client who is suspected of having a stroke. Which documen-
tation requires follow-up?
Onset of facial drooping at 1430
Alert and oriented to person but not oriented to place or time
Time of onset of stroke symptoms should be included in the
Onset of facial drooping at 1430 client interview. All other assessments are part of the physical
assessment.
Right-sided grip stronger than left-sided grip

Stroke scale completed


The nurse is planning care for a client who has unilateral neglect
and left-sided paralysis after experiencing a thrombotic stroke.
The client will participate in therapies to prevent contractures.
Which goal of care should the nurse choose?
Preventing contractures is a good goal for a client with left-sided
The client will maintain bedrest.
paralysis and unilateral neglect. The client will be taught active
range-of-motion exercises and ambulate as able, so maintaining
The client will participate in therapies to prevent contractures.
bedrest is not appropriate. An appropriate goal for blood pressure
is within normal limits, rather than 40% of normal. There is no
The client will improve communication techniques.
indication that the client needs assistance with communication.
The client's blood pressure will remain within 40% of normal.
The nurse is observing the unlicensed assistive personnel (UAP)
helping a client with unilateral neglect of the right side perform
self-care. Which statement by the UAP requires an intervention
by the nurse?
"When getting dressed, first put clothing on the left side."
"When getting dressed, first put clothing on the left side."
The client should be taught to dress the affected extremities first
"Use the left arm to bathe, brush teeth, comb hair, and eat." and then the unaffected extremities. This will enable the client to
dress herself with minimal assistance. The other options are all
"The occupational therapist will teach you how to promote upper appropriate instructions to teach the client to perform self-care.
extremity strength."

"The occupational therapist will assist you in learning to walk


using a walker."
After performing swallowing studies for a client recovering from
a stroke, the speech therapist recommends a pureed diet and
Carefully monitoring for coughing after giving the client a thick-
honey-thick liquids. Which is a priority for the nurse?
ened beverage
Calling the healthcare provider about the results
Maintaining client safety is a priority when feeding for the first
time. While all the answer options are appropriate, the priority is to
Ordering a pureed diet
assess the client for coughing when eating or drinking a thickened
liquid.
Documenting the results of the swallowing studies

4/5
Stroke NCLEX Practice Questions
Study online at https://quizlet.com/_9l9w70
Carefully monitoring for coughing after giving the client a thick-
ened beverage

5/5
STROKE- NCLEX
Study online at https://quizlet.com/_54u2bo
Difficulty swallowing
The nurse is caring for a patient with a history of transient is-
The patient's inability to swallow without difficulty would cause
chemic attacks (TIAs) and moderate carotid stenosis who has
the nurse the most concern. Difficulty in swallowing, hoarseness
undergone a carotid endarterectomy. Which of the following post-
or other signs of cranial nerve dysfunction must be assessed.
operative findings would cause the nurse the most concern?
The nurse focuses on assessment of the following cranial nerves:
facial (VII), vagus (X), spinal accessory (XI), and hypoglossal
(XII). Some edema in the neck after surgery is expected; how-
a) Blood pressure (BP): 128/86 mm Hg
ever, extensive edema and hematoma formation can obstruct the
airway. Emergency airway supplies, including those needed for
b) Neck pain: 3/10 (0 to 10 pain scale)
a tracheostomy, must be available. The patient's neck pain and
mild BP elevation need addressing but would not cause the nurse
c) Mild neck edema
the most concern. Hypotension is avoided to prevent cerebral
ischemia and thrombosis. Uncontrolled hypertension may precip-
d) Difficulty swallowing
itate cerebral hemorrhage, edema, hemorrhage at the surgical
incision, or disruption of the arterial reconstruction.
An emergency department nurse is interviewing a client with signs
of an ischemic stroke that began 2 hours ago. The client reports
that she had a cholecystectomy 6 weeks ago and is taking digoxin,
coumadin, and labetelol. This client is not eligible for thrombolytic
She is taking coumadin.
therapy for which of the following reasons?
To be eligible for thrombolytic therapy, the client cannot be taking
coumadin. Initiation of thrombolytic therapy must be within 3 hours
a) She is not within the treatment time window.
in clients with ischemic stroke. The client is not eligible for throm-
bolytic therapy if she has had surgery within 14 days. Digoxin and
b) She had surgery 6 weeks ago.
labetelol do not prohibit thrombolytic therapy.
c) She is taking digoxin.

d) She is taking coumadin.


Which disturbance results in loss of half of the visual field?
Homonymous hemianopsia
a) Anisocoria
Homonymous hemianopsia (loss of half of the visual field) may
occur from stroke and may be temporary or permanent. Double
b) Homonymous hemianopsia
vision is documented as diplopia. Nystagmus is ocular bobbing
and may be seen in multiple sclerosis. Anisocoria is unequal
c) Nystagmus
pupils.
d) Diplopia
A client with a cerebrovascular accident (CVA) is having difficulty
with eating food on the plate. Which is the best nursing action to
Perform a vision field assessment.
be taken?
The nurse should perform a vision field assessment to evaluate
the client forhemianopia. This finding could indicate damage to the
a) Reposition the tray and plate.
visual area of the brain as a result of evolving CVA. Repositioning
the tray and assisting with feeding would not be the best nursing
b) Perform a vision field assessment.
action until new finding has been evaluated. Hemianopia can be
associated with a CVA but, when presenting as a new finding,
c) Know this is a normal finding for CVA.
should be evaluated and reported immediately. (less)
d) Assist the client with feeding.
A client is hospitalized when they present to the Emergency
Department with right-sided weakness. Within 6 hours of being
Transient ischemic attack
admitted, the neurologic deficits had resolved and the client was
back to their presymptomatic state. The nurse caring for the client
A transient ischemic attack (TIA) is a sudden, brief attack of neu-
knows that the probable cause of the neurologic deficit was what?
rologic impairment caused by a temporary interruption in cerebral
blood flow. Symptoms may disappear within 1 hour; some contin-
ue for as long as 1 day. When the symptoms terminate, the client
a) Cerebral aneurysm

1 / 19
STROKE- NCLEX
Study online at https://quizlet.com/_54u2bo
b) Transient ischemic attack
resumes his or her presymptomatic state. The symptoms do not
c) Left-sided stroke
describe a left- or right-sided stroke or a cerebral aneurysm.
d) Right-sided stroke
Which of the following terms refer to the failure to recognize
familiar objects perceived by the senses?
Agnosia

Auditory agnosia is failure to recognize significance of sounds.


a) Agnosia
Agraphia refers to disturbances in writing intelligible words. Aprax-
ia refers to inability to perform previously learned purposeful motor
b) Perseveration
acts on a voluntary basis. Perseveration is the continued and
automatic repetition of an activity, word, or phrase that is no longer
c) Apraxia
appropriate.
d) Agraphia
During a class on stroke, a junior nursing student asks what the
clinical manifestations of stroke are. What would be the instruc-
tor's best answer? "Clinical manifestations of a stroke depend on the area of the
cortex, the affected hemisphere, the degree of blockage, and the
availability of collateral circulation."
a) "Clinical manifestations of a stroke depend on the area of the
cortex, the affected hemisphere, the degree of blockage, and the Clinical manifestations following a stroke are highly variable and
availability of collateral circulation." depend on the area of the cerebral cortex and the affected hemi-
sphere, the degree of blockage (total, partial), and the presence or
b) "Clinical manifestations of a stroke generally include aphasia, absence of adequate collateral circulation. (Collateral circulation
one-sided flaccidity, and trouble swallowing." is circulation formed by smaller blood vessels branching off from
or near larger occluded vessels.) Clinical manifestations of a
c) "Clinical manifestations of a stroke depend on how quickly the stroke do not depend on the cardiovascular health of the client
clot can be dissolved." or how quickly the clot can be dissolved. Clinical manifestations
of a stroke are not "general" but individual.
d) "Clinical manifestations of a stroke are highly variable, de-
pending on the cardiovascular health of the client."
When communicating with a client who has sensory (receptive)
aphasia, the nurse should: use short, simple sentences.

Although sensory aphasia allows the client to hear words, it im-


a) speak loudly and articulate clearly. pairs the ability to comprehend their meaning. The nurse should
use short, simple sentences to promote comprehension. Allowing
b) allow time for the client to respond. time for the client to respond might be helpful but is less important
than simplifying the communication. Because the client's hearing
c) give the client a writing pad. isn't affected, speaking loudly isn't necessary. A writing pad is
helpful for clients with expressive, not receptive, aphasia.
d) use short, simple sentences.
The nurse is providing information about strokes to a community
group. Which of the following would the nurse identify as the Weakness on one side of the body and difficulty with speech
primary initial symptoms of an ischemic stroke?
The main presenting symptoms for an ischemic stroke are numb-
ness or weakness of the face, arm, or leg, especially on one side
a) Footdrop and external hip rotation of the body; confusion or change in mental status; and trouble
speaking or understanding speech. Severe headache, vomiting,
b) Vomiting and seizures early change in level of consciousness, and seizures are early
signs of a hemorrhagic stroke. Footdrop and external hip rotation
c) Severe headache and early change in level of consciousness are things that can occur if a stroke victim is not turned or posi-
tioned correctly.
d) Weakness on one side of the body and difficulty with speech
Which of the following insults or abnormalities can cause an
ischemic stroke? Cocaine use

Cocaine is a potent vasoconstrictor and may result in a life-threat-


a) Arteriovenous malformation
2 / 19
STROKE- NCLEX
Study online at https://quizlet.com/_54u2bo

b) Intracerebral aneurysm rupture ening reaction, even with the individual's first use of the drug.
Arteriovenous malformations are associated with hemorrhagic
c) Cocaine use strokes. Trauma is associated with hemorrhagic strokes. Intrac-
erebral aneurysm rupture is associated with hemorrhagic strokes.
d) Trauma
A nurse is working with a student nurse who is caring for a client
with an acute bleeding cerebral aneurysm. Which action by the
student nurse requires further intervention? Keeping the client in one position to decrease bleeding

The student nurse shouldn't keep the client in one position. She
a) Maintaining the client in a quiet environment should carefully reposition the client often (at least every hour).
The client needs to be positioned so that a patent airway can
b) Keeping the client in one position to decrease bleeding be maintained. Fluid administration must be closely monitored
to prevent complications such as increased intracranial pressure.
c) Positioning the client to prevent airway obstruction The client must be maintained in a quiet environment to decrease
the risk of rebleeding.
d) Administering I.V. fluid as ordered and monitoring the client for
signs of fluid volume excess
Establishing eye contact
The nurse is caring for a patient with aphasia. Which of the
The following strategies should be used by the nurse to encourage
following strategies will the nurse use to facilitate communication
communication with a patient with aphasia: face the patient and
with the patient?
establish eye contact, speak in your usual manner and tone, use
short phrases, and pause between phrases to allow the patient
time to understand what is being said; limit conversation to prac-
a) Speaking in complete sentences
tical and concrete matters; use gestures, pictures, objects, and
writing; and as the patient uses and handles an object, say what
b) Speaking loudly
the object is. It helps to match the words with the object or action,
be consistent in using the same words and gestures each time
c) Avoiding the use of hand gestures
you give instructions or ask a question, and keep extraneous
noises and sounds to a minimum. Too much background noise
d) Establishing eye contact
can distract the patient or make it difficult to sort out the message
being spoken. (less)
While providing information to a community group, the nurse tells
Severe headache and early change in level of consciousness
them the primary initial symptoms of a hemorrhagic stroke are:
The main presenting symptoms for ischemic stroke are numbness
or weakness of the face, arm, or leg, especially on one side
a) Footdrop and external hip rotation
of the body, confusion or change in mental status, and trouble
speaking or understanding speech. Severe headache, vomiting,
b) Severe headache and early change in level of consciousness
early change in level of consciousness, and seizures are early
signs of a hemorrhagic stroke. Footdrop and external hip rotation
c) Weakness on one side of the body and difficulty with speech
can occur if a stroke victim is not turned or positioned correctly.
(less)
d) Confusion or change in mental status
A client is admitted with weakness, expressive aphasia, and right
hemianopia. The brain MRI reveals an infarct. The nurse under-
stands these symptoms to be suggestive of which of the following
findings? Left-sided cerebrovascular accident (CVA)

When the infarct is on the left side of the brain, the symptoms
a) Left-sided cerebrovascular accident (CVA) are likely to be on the right, and the speech is more likely to
be involved. If the MRI reveals an infarct, TIA is no longer the
b) Right-sided cerebrovascular accident (CVA) diagnosis. There is not enough information to determine if the
stroke is still evolving or is complete.
c) Transient ischemic attack (TIA)

d) Completed Stroke
A nurse is caring for a client who has returned to his room after a
carotid endarterectomy. Which action should the nurse take first?

3 / 19
STROKE- NCLEX
Study online at https://quizlet.com/_54u2bo
Ask the client if he has trouble breathing.
a) Take the client's blood pressure.
The nurse should first assess the client's breathing. A complica-
b) Ask the client if he has a headache.
tion of a carotid endarterectomy is an incisional hematoma, which
could compress the trachea causing breathing difficulty for the
c) Ask the client if he has trouble breathing.
client. Although the other measures are important actions, they
aren't the nurse's top priority.
d) Place antiembolism stockings on the client.
The nurse is caring for a patient with dysphagia. Which of the
following interventions would be contraindicated while caring for
this patient?
Placing food on the affected side of mouth

a) Allowing ample time to eat Interventions for dysphagia include placing food on the unaffected
side of the mouth, allowing ample time to eat, assisting the patient
b) Assisting the patient with meals with meals, and testing the patient's gag reflex prior to offering
food or fluids.
c) Testing the gag reflex prior to offering food or fluids

d) Placing food on the affected side of mouth


A 64-year-old client reports symptoms consistent with a transient
ischemic attack (TIA) to the physician in the emergency depart-
Impaired cerebral circulation
ment. After completing ordered diagnostic tests, the physician
indicates to the client what caused the symptoms that brought
TIAs result from impaired blood circulation in the brain, which
him to the hospital. What is the origin of the client's symptoms?
can be caused by atherosclerosis and arteriosclerosis, cardiac
disease, or diabetes. The symptoms of a TIA are the result of
impaired blood circulation in the brain, which may have been
a) Hypertension
caused by cardiac disease. The symptoms of a TIA are the result
of impaired blood circulation in the brain, which may have been
b) Cardiac disease
caused by diabetes. The symptoms of a TIA are the result of
impaired blood circulation in the brain, which may have been
c) Diabetes insipidus
caused by hypertension.
d) Impaired cerebral circulation
A client is admitted to the intensive care unit (ICU) with a diag-
nosis of cerebrovascular accident (CVA). Which assessment by
the nurse provides the most significant finding in differentiating
A unit of fresh frozen plasma is infusing.
between ischemic and hemorrhagic strokes?
FFP is usedin the treatment of clotting deficiencies as seen with
over dose of anticoagulants and would indicate a hemorrhagic
a) Oropharyngeal suctioning as needed.
stroke. Neuro checks ordered every 2 hours does not differentiate
between types of strokes. Focal seizures can occur with any stroke
b) Kepprais ordered for treatment of focal seizures.
and would not differentiate. Suctioning is a nursing action taken to
maintain airway and does not indicate a specific type of stroke.
c) A unit of fresh frozen plasma is infusing.

d) Neurological checks are ordered every 2 hours.


Establishing eye contact
The nurse is caring for a patient with aphasia. Which of the
following strategies will the nurse use to facilitate communication
The following strategies should be used by the nurse to encourage
with the patient?
communication with a patient with aphasia: face the patient and
establish eye contact, speak in your usual manner and tone, use
short phrases, and pause between phrases to allow the patient
a) Avoiding the use of hand gestures
time to understand what is being said; limit conversation to prac-
tical and concrete matters; use gestures, pictures, objects, and
b) Establishing eye contact
writing; and as the patient uses and handles an object, say what
the object is. It helps to match the words with the object or action,
c) Speaking in complete sentences
be consistent in using the same words and gestures each time
you give instructions or ask a question, and keep extraneous
d) Speaking loudly
noises and sounds to a minimum. Too much background noise

4 / 19
STROKE- NCLEX
Study online at https://quizlet.com/_54u2bo
can distract the patient or make it difficult to sort out the message
being spoken. (less)
The provider diagnoses the patient as having had an ischemic
stroke. The etiology of an ischemic stroke would include which of
the following?
Cardiogenic emboli
a) Cerebral aneurysm
Aneurysms, hemorrhages, and malformations are all examples of
a hemorrhagic stroke. An embolism can block blood flow, leading
b) Cardiogenic emboli
to ischemia.
c) Intracerebral hemorrhage

d) Arteriovenous malformation
A patient is in the acute phase of an ischemic stroke. How long
does the nurse know that this phase may last?

1 to 3 days
a) Up to 1 week
The acute phase of an ischemic stroke may last 1 to 3 days, but
b) Up to 24 hours ongoing monitoring of all body systems is essential as long as the
patient requires care.
c) 1 to 3 days

d) Up to 2 weeks
Which of the following terms refer to the inability to perform
previously learned purposeful motor acts on a voluntary basis?
Apraxia

Verbal apraxia refers to difficulty in forming and organizing intel-


a) Agraphia
ligible words although the musculature is intact. Agnosia is failure
to recognize familiar objects perceived by the senses. Agraphia
b) Perseveration
refers to disturbances in writing intelligible words. Perseveration
is the continued and automatic repetition of an activity or word or
c) Agnosia
phrase that is no longer appropriate. (less)
d) Apraxia
A nurse is working with a student nurse who is caring for a client
with an acute bleeding cerebral aneurysm. Which action by the
student nurse requires further intervention? Keeping the client in one position to decrease bleeding

The student nurse shouldn't keep the client in one position. She
a) Maintaining the client in a quiet environment should carefully reposition the client often (at least every hour).
The client needs to be positioned so that a patent airway can
b) Positioning the client to prevent airway obstruction be maintained. Fluid administration must be closely monitored
to prevent complications such as increased intracranial pressure.
c) Keeping the client in one position to decrease bleeding The client must be maintained in a quiet environment to decrease
the risk of rebleeding.
d) Administering I.V. fluid as ordered and monitoring the client for
signs of fluid volume excess
A client has experienced an ischemic stroke that has damaged the
lower motor neurons of the brain. Which of the following deficits Lack of deep tendon reflexes
would the nurse expect during assessment?
Damage to the occipital lobe can result in visual agnosia, whereas
damage to the temporal lobe can cause auditory agnosia. If dam-
a) Limited attention span and forgetfulness age has occurred to the frontal lobe, learning capacity, memory, or
other higher cortical intellectual functions may be impaired. Such
b) Visual agnosia dysfunction may be reflected in a limited attention span, difficulties
in comprehension, forgetfulness, and lack of motivation. Damage
c) Auditory agnosia to the lower motor neurons may cause decreased muscle tone,
flaccid muscle paralysis, and a decrease in or loss of reflexes.
d) Lack of deep tendon reflexes
5 / 19
STROKE- NCLEX
Study online at https://quizlet.com/_54u2bo
A nurse knows that, for a patient with an ischemic stroke, tPA is
contraindicated if the blood pressure reading is:

190 mm Hg/120 mm Hg
a) 170 mm Hg/105 mm Hg
Elevated blood pressure (systolic >185; diastolic >110 mm Hg) is
b) 185 mm Hg/110 mm Hg
a contraindication to tPA.
c) 190 mm Hg/120 mm Hg

d) 175 mm Hg/100 mm Hg
A patient who has suffered a stroke begins having complications
regarding spasticity in the lower extremity. What ordered medica-
Lioresal (Baclofen)
tion does the nurse administer to help alleviate this problem?
Spasticity, particularly in the hand, can be a disabling complica-
tion after stroke. Botulinum toxin type A injected intramuscularly
a) Pregabalin (Lyrica)
into wrist and finger muscles has been shown to be effective in
reducing this spasticity (although the effect is temporary, typically
b) Diphenhydramine (Benadryl)
lasting 2 to 4 months) (Teasell, Foley, Pereira, et al., 2012). Other
treatments for spasticity may include stretching, splinting, and oral
c) Heparin
medications such as baclofen (Lioresal). (less)
d) Lioresal (Baclofen)
A patient diagnosed with a stroke is ordered to receive warfarin
(Coumadin). Later, the nurse learns that the warfarin is contraindi-
cated and the order is canceled. The nurse knows that the best
alternative medication to give is which of the following?
Aspirin
a) Ticlodipine (Ticlid)
If warfarin is contraindicated, aspirin is the best option, although
other medications may be used if both are contraindicated.
b) Dipyridamole (Persantine)

c) Clopidogrel (Plavix)

d) Aspirin
Which of the following is the most common side effect of tissue
plasminogen activator (tPA)?
Bleeding

a) Increased intracranial pressure (ICP) Bleeding is the most common side effect of tPA. The patient
is closely monitored for bleeding (at IV insertion sites, gums,
b) Hypertension urine/stools, and intracranially by assessing changes in level of
consciousness). Headache, increased ICP, and hypertension are
c) Headache not side effects of tPA.

d) Bleeding
Which of the following antiseizure medication has been found to
be effective for post-stroke pain?

Lamotrigine (Lamictal)
a) Carbamazepine (Tegretol)
The antiseizure medication lamotrigine (Lamictal) has been found
b) Lamotrigine (Lamictal)
to be effective for post-stroke pain.
c) Topiramate (Topamax)

d) Phenytoin (Dilantin)
The nurse practitioner advises a patient who is at high risk for a
stroke to be vigilant in his medication regime, to maintain a healthy
weight, and to adopt a reasonable exercise program. This advice
6 / 19
STROKE- NCLEX
Study online at https://quizlet.com/_54u2bo
is based on research data that shows the most important risk
factor for stroke is:

Hypertension
a) Dyslipidemia
Hypertension is the most modifiable risk factor for either ischemic
b) Obesity or hemorrhagic stroke. Unfortunately, it remains under-recognized
and undertreated in most communities.
c) Hypertension

d) Smoking
While providing information to a community group, the nurse tells
Severe headache and early change in level of consciousness
them the primary initial symptoms of a hemorrhagic stroke are:
The main presenting symptoms for ischemic stroke are numbness
or weakness of the face, arm, or leg, especially on one side
a) Footdrop and external hip rotation
of the body, confusion or change in mental status, and trouble
speaking or understanding speech. Severe headache, vomiting,
b) Severe headache and early change in level of consciousness
early change in level of consciousness, and seizures are early
signs of a hemorrhagic stroke. Footdrop and external hip rotation
c) Confusion or change in mental status
can occur if a stroke victim is not turned or positioned correctly.
(less)
d) Weakness on one side of the body and difficulty with speech
A client is admitted with weakness, expressive aphasia, and right
hemianopia. The brain MRI reveals an infarct. The nurse under-
stands these symptoms to be suggestive of which of the following
findings? Left-sided cerebrovascular accident (CVA)

When the infarct is on the left side of the brain, the symptoms
a) Left-sided cerebrovascular accident (CVA) are likely to be on the right, and the speech is more likely to
be involved. If the MRI reveals an infarct, TIA is no longer the
b) Completed Stroke diagnosis. There is not enough information to determine if the
stroke is still evolving or is complete.
c) Transient ischemic attack (TIA)

d) Right-sided cerebrovascular accident (CVA)


A physician orders several drugs for a client with hemorrhagic
stroke. Which drug order should the nurse question? Heparin sodium

Administering heparin, an anticoagulant, could increase the


a) Phenytoin (Dilantin) bleeding associated with hemorrhagic stroke. Therefore, the nurse
should question this order to prevent additional hemorrhage in
b) Methyldopa (Aldomet) the brain. In a client with hemorrhagic stroke, the physician may
use dexamethasone to decrease cerebral edema and pressure;
c) Heparin sodium methyldopa, to reduce blood pressure; and phenytoin, to prevent
seizures.
d) Dexamethasone (Decadron)
A patient presents to the emergency room with complaints of
having an "exploding headache" for the last 2 hours. The patient
is immediately seen by a triage nurse who suspects the patient is
experiencing a stroke. Which of the following is a possible cause
based on the characteristic symptom?
Cerebral aneurysm

a) Cerebral aneurysm A cerebral aneurysm is a type of hemorrhagic stroke that is


characterized by an exploding headache.
b) Cardiogenic emboli

c) Large artery thrombosis

d) Small artery thrombosis

7 / 19
STROKE- NCLEX
Study online at https://quizlet.com/_54u2bo
Which of the following is accurate regarding a hemorrhagic
stroke?
Main presenting symptom is an "exploding headache."
a) It is caused by a large-artery thrombosis.
One of hemorrhagic stroke's main presenting symptom is an
"exploding headache." In ischemic stroke, functional recovery
b) One of the main presenting symptoms is numbness or weak-
usually plateaus at 6 months; it may be caused by a large artery
ness of the face.
thrombosis and may have a presenting symptoms of numbness
or weakness of the face
c) Main presenting symptom is an "exploding headache."

d) Functional recovery usually plateaus at 6 months.


A 73-year-old client is visiting the neurologist. The client reports
light-headedness, speech disturbance, and left-sided weakness
that have lasted for several hours. In the examination, an abnor-
mal sound is auscultated in an artery leading to the brain. What
Bruit
is the term for the auscultated discovery?
A neurologic examination during an attack reveals neurologic
deficits. Auscultation of the artery may reveal a bruit (abnormal
a) Atherosclerotic plaque
sound caused by blood flowing over a rough surface within one
or both carotid arteries). The term for the auscultated discovery is
b) TIA
bruit.
c) Diplopia

d) Bruit
Which of the following statements reflect nursing management of
Encourage the patient to repeat sounds of the alphabet
the patient with expressive aphasia?
Nursing management of the patient with expressive aphasia in-
cludes encouraging the patient to repeat sounds of the alphabet.
a) Frequently reorient the patient to time, place, and situation
Nursing management of the patient with global aphasia includes
speaking clearly to the patient in simple sentences and using ges-
b) Speak clearly to the patient in simple sentences, use gestures
tures or pictures when able. Nursing management of the patient
or pictures when able
with receptive aphasia includes speaking slowing and clearly to
assist the patient in forming the sounds. Nursing management of
c) Speak slowly and clearly to assist the patient in forming the
the patient with cognitive deficits, such as memory loss, includes
sounds
frequently reorienting the patient to time, place, and situation.
(less)
d) Encourage the patient to repeat sounds of the alphabet
Which of the following is the initial diagnostic test for a stroke?

Noncontrast CT scan
a) Noncontrast CT scan
The initial diagnostic test for a stroke is a nonconstrast CT scan
b) Transcranial Doppler studies performed emergently to determine if the event is ischemic or
hemorrhagic. Further diagnostics include a carotid Doppler, ECG,
c) ECG and a transcranial Doppler. (less)

d) Carotid Doppler
A patient has been diagnosed as having global aphasia. The
Form words that are understandable or comprehend the spoken
nurse recognizes that the patient will be unable to do which of the
word
following actions?
Global aphasia is a combination of expressive and receptive
aphasia and presents tremendous challenge to the nurse to
a) Form words that are understandable
effectively communicate with the patient. In receptive aphasia,
the patient is unable to form words that are understandable. In
b) Speak at all
expressive aphasia, the patient is unable to form words that are
understandable. The patient who is unable to speak at all is
c) Form words that are understandable or comprehend the spo-
referred to as mute.
ken word

8 / 19
STROKE- NCLEX
Study online at https://quizlet.com/_54u2bo

d) Comprehend the spoken word


A patient is admitted via ambulance to the emergency room of
a stroke center at 1:30 p.m. with symptoms that the patient said
began at 1:00 p.m. Within 1 hour, an ischemic stroke had been
confirmed and the doctor ordered tPA. The nurse knows to give
this drug no later than what time?
4:00 p.m.

Tissue plasminogen activator (tPA) must be given within 3 hours


a) 5:30 p.m.
after symptom onset. Therefore, since symptom onset was 1:00
pm, the window of opportunity ends at 4:00 pm.
b) 3:00 p.m.

c) 4:00 p.m.

d) 2:30 p.m.
The nurse is providing diet-related advice to a male patient fol-
lowing a cerebrovascular accident (CVA). The patient wants to
Provide thickened commercial beverages and fortified cooked
minimize the volume of food and yet meet all nutritional elements.
cereals.
Which of the following suggestions should the nurse give to the
patient about controlling the volume of food intake?
Patients with CVA or other cerebrovascular disorders should lose
weight and therefore should minimize their volume of food con-
sumption. To ensure this, the nurse may provide thickened com-
a) Provide a high-fat diet.
mercial beverages, fortified cooked cereals, or scrambled eggs.
Patients should avoid eating high-fat foods, and serving foods hot
b) Include dry or crisp foods and chewy meats.
or tepid will not minimize the volume consumed by the patient.
Foods such as peanut butter, bread, tart foods, dry or crisp foods,
c) Always serve hot or tepid foods.
and chewy meats should also be avoided because they cause
choking.
d) Provide thickened commercial beverages and fortified cooked
cereals.
A client is receiving an I.V. infusion of mannitol (Osmitrol) after
undergoing intracranial surgery to remove a brain tumor. To de-
termine whether this drug is producing its therapeutic effect, the
nurse should consider which finding most significant?
Increased urine output

The therapeutic effect of mannitol is diuresis, which is confirmed


a) Elevated blood pressure
by an increased urine output. A decreased LOC and elevated
blood pressure may indicate lack of therapeutic effectiveness. A
b) Decreased level of consciousness (LOC)
decreased heart rate doesn't indicate that mannitol is effective.
c) Increased urine output

d) Decreased heart rate


Which of the following is a contraindication for the administration
of tissue plasminogen activator (t-PA)?
Intracranial hemorrhage
a) Systolic blood pressure less than or equal to 185 mm Hg
Intracranial hemorrhage, neoplasm, or aneurysm is a contraindi-
cation to t-PA. Clinical diagnosis of ischemic stroke, age 18 years
b) Ischemic stroke
of age or older, and a systolic BP less than or equal to 185 mm
Hg are eligibility criteria. (less)
c) Intracranial hemorrhage

d) Age 18 years of age or older


1. A patient has had an ischemic stroke and has been admitted
to the medical unit. What action should the nurse perform to best
prevent joint deformities? C) Place a pillow in the axilla when there is limited external
A) Place the patient in the prone position for 30 minutes/day. rotation.
B) Assist the patient in acutely flexing the thigh to promote move-
ment.
9 / 19
STROKE- NCLEX
Study online at https://quizlet.com/_54u2bo
C) Place a pillow in the axilla when there is limited external
rotation.
D) Place patient's hand in pronation.
A patient diagnosed with transient ischemic attacks (TIAs) is
scheduled for a carotid endarterectomy. The nurse explains that
this procedure will be done for what purpose?
A) To decrease cerebral edema C) To remove atherosclerotic plaques blocking cerebral flow
B) To prevent seizure activity that is common following a TIA
C) To remove atherosclerotic plaques blocking cerebral flow
D) To determine the cause of the TIA
The nurse is discharging home a patient who suffered a stroke.
He has a flaccid right arm and leg and is experiencing problems
with urinary incontinence. The nurse makes a referral to a home
health nurse because of an awareness of what common patient
response to a change in body image? C) Depression
A) Denial
B) Fear
C) Depression
D) Disassociation
When caring for a patient who had a hemorrhagic stroke, close
monitoring of vital signs and neurologic changes is imperative.
What is the earliest sign of deterioration in a patient with a hem-
orrhagic stroke of which the nurse should be aware?
B) Alteration in level of consciousness (LOC)
A) Generalized pain
B) Alteration in level of consciousness (LOC)
C) Tonic-clonic seizures
D) Shortness of breath
The nurse is performing stroke risk screenings at a hospital open
house. The nurse has identified four patients who might be at
risk for a stroke. Which patient is likely at the highest risk for a
hemorrhagic stroke?
B) White male, age 60, with history of uncontrolled hypertension
A) White female, age 60, with history of excessive alcohol intake
B) White male, age 60, with history of uncontrolled hypertension
C) Black male, age 60, with history of diabetes
D) Black male, age 50, with history of smoking
A patient who just suffered a suspected ischemic stroke is brought
to the ED by ambulance. On what should the nurse's primary
assessment focus?
A) Cardiac and respiratory status A) Cardiac and respiratory status
B) Seizure activity
C) Pain
D) Fluid and electrolyte balance
A patient with a cerebral aneurysm exhibits signs and symptoms
of an increase in intracranial pressure (ICP). What nursing inter-
vention would be most appropriate for this patient?
A) Range-of-motion exercises to prevent contractures D) Absolute bed rest in a quiet, nonstimulating environment
B) Encouraging independence with ADLs to promote recovery
C) Early initiation of physical therapy
D) Absolute bed rest in a quiet, nonstimulating environment
A patient recovering from a stroke has severe shoulder pain from
subluxation of the shoulder and is being cared for on the unit. To
prevent further injury and pain, the nurse caring for this patient is
aware of what principle of care?
A) The patient should be fitted with a cast because use of a sling D) The patient should be taught to interlace fingers, place palms
should be avoided due to adduction of the affected shoulder. together, and slowly bring scapulae forward to avoid excessive
B) Elevation of the arm and hand can lead to further complications force to shoulder.
associated with edema.
C) Passively exercising the affected extremity is avoided in order
to minimize pain.
D) The patient should be taught to interlace fingers, place palms
10 / 19
STROKE- NCLEX
Study online at https://quizlet.com/_54u2bo
together, and slowly bring scapulae forward to avoid excessive
force to shoulder.
The patient has been diagnosed with aphasia after suffering a
stroke. What can the nurse do to best make the patient's atmos-
phere more conducive to communication?
A) Provide a board of commonly used needs and phrases. A) Provide a board of commonly used needs and phrases.
B) Have the patient speak to loved ones on the phone daily.
C) Help the patient complete his or her sentences.
D) Speak in a loud and deliberate voice to the patient.
The nurse is assessing a patient with a suspected stroke. What
assessment finding is most suggestive of a stroke?
A) Facial droop
A) Facial droop
B) Dysrhythmias
C) Periorbital edema
D) Projectile vomiting
The nurse is caring for a patient diagnosed with an ischemic
stroke and knows that effective positioning of the patient is impor-
tant. Which of the following should be integrated into the patient's
plan of care?
A) The patient's hip joint should be maintained in a flexed position. C) The patient should be placed in a prone position for 15 to 30
B) The patient should be in a supine position unless ambulating. minutes several times a day.
C) The patient should be placed in a prone position for 15 to 30
minutes several times a day.
D) The patient should be placed in a Trendelenberg position two
to three times daily to promote cerebral perfusion.
A patient has been admitted to the ICU after being recently
diagnosed with an aneurysm and the patient's admission orders
include specific aneurysm precautions. What nursing action will
the nurse incorporate into the patient's plan of care?
B) Maintain the patient on complete bed rest.
A) Elevate the head of the bed to 45 degrees.
B) Maintain the patient on complete bed rest.
C) Administer enemas when the patient is constipated.
D) Avoid use of thigh-high elastic compression stockings.
A nurse is caring for a patient diagnosed with a hemorrhagic
stroke. When creating this patient's plan of care, what goal should
be prioritized?
A) Prevent complications of immobility. B) Maintain and improve cerebral tissue perfusion.
B) Maintain and improve cerebral tissue perfusion.
C) Relieve anxiety and pain.
D) Relieve sensory deprivation.
The nurse is preparing health education for a patient who is being
discharged after hospitalization for a hemorrhagic stroke. What
content should the nurse include in this education?
A) Mild, intermittent seizures can be expected. C) Take antihypertensive medication as ordered.
B) Take ibuprofen for complaints of a serious headache.
C) Take antihypertensive medication as ordered.
D) Drowsiness is normal for the first week after discharge.
A patient diagnosed with a cerebral aneurysm reports a severe
headache to the nurse. What action is a priority for the nurse?
A) Sit with the patient for a few minutes.
D) Call the physician immediately.
B) Administer an analgesic.
C) Inform the nurse-manager.
D) Call the physician immediately.
A patient is brought by ambulance to the ED after suffering what
the family thinks is a stroke. The nurse caring for this patient is
aware that an absolute contraindication for thrombolytic therapy
A) Evidence of hemorrhagic stroke
is what?
A) Evidence of hemorrhagic stroke
B) Blood pressure of 180/110 mm Hg

11 / 19
STROKE- NCLEX
Study online at https://quizlet.com/_54u2bo
C) Evidence of stroke evolution
D) Previous thrombolytic therapy within the past 12 months
When caring for a patient who has had a stroke, a priority is
reduction of ICP. What patient position is most consistent with this
goal?
A) Head turned slightly to the right side B) Elevation of the head of the bed
B) Elevation of the head of the bed
C) Position changes every 15 minutes while awake
D) Extension of the neck
A patient who suffered an ischemic stroke now has disturbed
sensory perception. What principle should guide the nurse's care
of this patient?
A) The patient should be approached on the side where visual
perception is intact.
A) The patient should be approached on the side where visual
B) Attention to the affected side should be minimized in order to
perception is intact.
decrease anxiety.
C) The patient should avoid turning in the direction of the defective
visual field to minimize shoulder subluxation.
D) The patient should be approached on the opposite side of
where the visual perception is intact to promote recovery.
What should be included in the patient's care plan when estab-
lishing an exercise program for a patient affected by a stroke?
A) Schedule passive range of motion every other day.
B) Keep activity limited, as the patient may be over stimulated. D) Exercise the affected extremities passively four or five times a
C) Have the patient perform active range-of-motion (ROM) exer- day.
cises once a day.
D) Exercise the affected extremities passively four or five times a
day.
A female patient is diagnosed with a right-sided stroke. The patient
is now experiencing hemianopsia. How might the nurse help the
patient manage her potential sensory and perceptional difficul-
ties?
D) Place the patient's extremities where she can see them.
A) Keep the lighting in the patient's room low.
B) Place the patient's clock on the affected side.
C) Approach the patient on the side where vision is impaired.
D) Place the patient's extremities where she can see them.
The public health nurse is planning a health promotion campaign
that reflects current epidemiologic trends. The nurse should know
that hemorrhagic stroke currently accounts for what percentage
of total strokes in the United States?
D) 13%
A) 43%
B) 33%
C) 23%
D) 13%
A patient who has experienced an ischemic stroke has been
admitted to the medical unit. The patient's family in adamant that
she remain on bed rest to hasten her recovery and to conserve
energy. What principle of care should inform the nurse's response
to the family?
A) The patient should mobilize as soon as she is physically able.
A) The patient should mobilize as soon as she is physically able.
B) To prevent contractures and muscle atrophy, bed rest should
not exceed 4 weeks.
C) The patient should remain on bed rest until she expresses a
desire to mobilize.
D) Lack of mobility will greatly increase the patient's risk of stroke
recurrence.
A patient has recently begun mobilizing during the recovery from
an ischemic stroke. To protect the patient's safety during mobi-
lization, the nurse should perform what action?
A) Support the patient's full body weight with a waist belt during
12 / 19
STROKE- NCLEX
Study online at https://quizlet.com/_54u2bo
ambulation.
B) Have a colleague follow the patient closely with a wheelchair.
C) Avoid mobilizing the patient in the early morning or late
B) Have a colleague follow the patient closely with a wheelchair.
evening.
D) Ensure that the patient's family members do not participate in
mobilization.
A patient diagnosed with a hemorrhagic stroke has been admitted
to the neurologic ICU. The nurse knows that teaching for the
patient and family needs to begin as soon as the patient is settled
on the unit and will continue until the patient is discharged. What
will family education need to include?
C) How to correctly modify the home environment
A) How to differentiate between hemorrhagic and ischemic stroke
B) Risk factors for ischemic stroke
C) How to correctly modify the home environment
D) Techniques for adjusting the patient's medication dosages at
home
After a subarachnoid hemorrhage, the patient's laboratory results
indicate a serum sodium level of less than 126 mEq/L. What is the
nurse's most appropriate action?
A) Administer a bolus of normal saline as ordered. D) Prepare to administer 3% NaCl by IV as ordered.
B) Prepare the patient for thrombolytic therapy as ordered.
C) Facilitate testing for hypothalamic dysfunction.
D) Prepare to administer 3% NaCl by IV as ordered.
A community health nurse is giving an educational presentation
about stroke and heart disease at the local senior citizens center.
What nonmodifiable risk factor for stroke should the nurse cite?
A) Female gender C) Advanced age
B) Asian American race
C) Advanced age
D) Smoking
A family member brings the patient to the clinic for a follow-up visit
after a stroke. The family member asks the nurse what he can do
to decrease his chance of having another stroke. What would be
the nurse's best answer?
B) "Stop smoking as soon as possible."
A) "Have your heart checked regularly."
B) "Stop smoking as soon as possible."
C) "Get medication to bring down your sodium levels."
D) "Eat a nutritious diet."
The nurse is reviewing the medication administration record of a
female patient who possesses numerous risk factors for stroke.
Which of the woman's medications carries the greatest potential
for reducing her risk of stroke?
C) Aspirin 81 mg PO o.d.
A) Naproxen 250 PO b.i.d.
B) Calcium carbonate 1,000 mg PO b.i.d.
C) Aspirin 81 mg PO o.d.
D) Lorazepam 1 mg SL b.i.d. PRN
A nurse in the ICU is providing care for a patient who has been
admitted with a hemorrhagic stroke. The nurse is performing
frequent neurologic assessments and observes that the patient is
becoming progressively more drowsy over the course of the day.
What is the nurse's best response to this assessment finding?
A) Report this finding to the physician as an indication of de-
D) Report this to the physician as a possible sign of clinical
creased metabolism.
deterioration.
B) Provide more stimulation to the patient and monitor the patient
closely.
C) Recognize this as the expected clinical course of a hemor-
rhagic stroke.
D) Report this to the physician as a possible sign of clinical
deterioration.

13 / 19
STROKE- NCLEX
Study online at https://quizlet.com/_54u2bo
Following diagnostic testing, a patient has been admitted to the
ICU and placed on cerebral aneurysm precautions. What nursing
action should be included in patient's plan of care?
A) Supervise the patient's activities of daily living closely. A) Supervise the patient's activities of daily living closely.
B) Initiate early ambulation to prevent complications of immobility.
C) Provide a high-calorie, low-protein diet.
D) Perform all of the patient's hygiene and feeding.
A preceptor is discussing stroke with a new nurse on the unit. The
preceptor would tell the new nurse which cardiac dysrhythmia is
associated with cardiogenic embolic strokes?
A) Ventricular tachycardia B) Atrial fibrillation
B) Atrial fibrillation
C) Supraventricular tachycardia
D) Bundle branch block
The pathophysiology of an ischemic stroke involves the ischemic
cascade, which includes the following steps:
1. Change in pH
2. Blood flow decreases
3. A switch to anaerobic respiration
4. Membrane pumps fail
5. Cells cease to function
C) 236145
6. Lactic acid is generated

Put these steps in order in which they occur.


A) 635241
B) 352416
C) 236145
D) 162534
As a member of the stroke team, the nurse knows that thrombolyt-
ic therapy carries the potential for benefit and for harm. The nurse
should be cognizant of what contraindications for thrombolytic
therapy? Select all that apply. B) Recent intracranial pathology
A) INR above 1.0 D) Current anticoagulation therapy
B) Recent intracranial pathology E) Symptom onset greater than 3 hours prior to admission
C) Sudden symptom onset
D) Current anticoagulation therapy
E) Symptom onset greater than 3 hours prior to admission
After a major ischemic stroke, a possible complication is cerebral
edema. Nursing care during the immediate recovery period from
an ischemic stroke should include which of the following?
A) Positioning to avoid hypoxia A) Positioning to avoid hypoxia
B) Maximizing PaCO2
C) Administering hypertonic IV solution
D) Initiating early mobilization
The nurse is caring for a patient recovering from an ischemic
stroke. What intervention best addresses a potential complication
after an ischemic stroke?
A) Providing frequent small meals rather than three larger meals
B) Teaching the patient to perform deep breathing and coughing
B) Teaching the patient to perform deep breathing and coughing
exercises
exercises
C) Keeping a urinary catheter in situ for the full duration of recov-
ery
D) Limiting intake of insoluble fiber
During a patient's recovery from stroke, the nurse should be
aware of predictors of stroke outcome in order to help patients
and families set realistic goals. What are the predictors of stroke A) National Institutes of Health Stroke Scale (NIHSS) score
outcome? Select all that apply. C) LOC at time of admission
A) National Institutes of Health Stroke Scale (NIHSS) score E) Age
B) Race
C) LOC at time of admission

14 / 19
STROKE- NCLEX
Study online at https://quizlet.com/_54u2bo
D) Gender
E) Age
A nursing student is writing a care plan for a newly admitted pa-
tient who has been diagnosed with a stroke. What major nursing
diagnosis should most likely be included in the patient's plan of
care?
D) Disturbed sensory perception
A) Adult failure to thrive
B) Post-trauma syndrome
C) Hyperthermia
D) Disturbed sensory perception
When preparing to discharge a patient home, the nurse has met
with the family and warned them that the patient may exhibit
unexpected emotional responses. The nurse should teach the
family that these responses are typically a result of what cause?
A) Frustration around changes in function and communication
A) Frustration around changes in function and communication
B) Unmet physiologic needs
C) Changes in brain activity during sleep and wakefulness
D) Temporary changes in metabolism
A rehabilitation nurse caring for a patient who has had a stroke is
approached by the patient's family and asked why the patient has
to do so much for herself when she is obviously struggling. What
would be the nurse's best answer?
A) "We are trying to help her be as useful as she possibly can."
B) "The focus on care in a rehabilitation facility is to help the patient
B) "The focus on care in a rehabilitation facility is to help the
to resume as much self-care as possible."
patient to resume as much self-care as possible."
C) "We aren't here to care for her the way the hospital staff did;
we are here to help her get better so she can go home."
D) "Rehabilitation means helping patients do exactly what they
did before their stroke."
A patient with a new diagnosis of ischemic stroke is deemed to
be a candidate for treatment with tissue plasminogen activator
(t-PA) and has been admitted to the ICU. In addition to closely
monitoring the patient's cardiac and neurologic status, the nurse
monitors the patient for signs of what complication? C) Bleeding
A) Acute pain
B) Septicemia
C) Bleeding
D) Seizures
1. A nurse is communicating with a client who has aphasia after
having a stroke. Which action should the nurse take?
a. Use one long sentence to say everything that needs to be said.
d. Face the client and establish eye contact.
b. Keep the television on while she speaks.
c. Talk in a louder than normal voice.
d. Face the client and establish eye contact.
2. Face the patient and establish eye contact.
3. Speak in a clear, unhurried manner, and normal tone of voice.
4. Use short phrases, and pause between phrases to allow the
patient time to understand what is being said.
5. Limit conversation to practical and concrete matters.
6. Use gestures, pictures, objects, and writing.
Name two more "tips' in communicating with a patient with Apha-
7. As the patient uses and handles an object, say what the object
sia:
is. It helps to match the words with the object or action.
8. Be consistent in using the same words and gestures each time
you give instructions or ask a question.
9. Keep extraneous noises and sounds to a minimum. Too much
background noise can distract the patient or make it difficult to sort
out the message being spoken.
2. A client who has experienced an initial transient ischemic attack
(TIA) states: "I'm glad it wasn't anything serious." Which is the best
nursing response to this statement?
15 / 19
STROKE- NCLEX
Study online at https://quizlet.com/_54u2bo
a. "I sense that you are happy it was not a stroke".
b. "People who experience a TIA will develop a stroke".
d. "TIA is a warning sign. Let's talk about lowering your risks."
c. "TIA symptoms are short-lived and resolve within 24 hours".
d. "TIA is a warning sign. Let's talk about lowering your risks."
How long does the typical neurological deficit last with a TIA? less than 24 hrs, lasting 1-2 hrs
3. When caring for a patient who had a hemorrhagic stroke, close
monitoring of vital signs and neurologic changes is imperative.
What is the earliest sign of deterioration in a patient with a hem-
orrhagic stroke of which the nurse should be aware?
b. Alteration in level of consciousness (LOC)
a. Generalized pain
b. Alteration in level of consciousness (LOC)
c. Tonic-clonic seizures
d. Shortness of breath
What will alteration in level of consciousness (LOC) look like for
Drowsiness, slight slurring of speech, sluggish pupillary reaction
this patient? Name two symptoms.
4. When caring for a patient who has had a stroke, a priority is
reduction of ICP. What patient position is most consistent with this
goal?
a. Head turned slightly to the right side b. Elevation of the head of the bed
b. Elevation of the head of the bed
c. Position changes every 15 minutes while awake
d. Extension of the neck
Careful maintenance of cerebral hemodynamics to maintain cere-
TRUE
bral perfusion is extremely important after a stroke.
What is the cause of increased intracranial pressure (ICP)? CEREBRAL EDEMA
5. A patient diagnosed with an ischemic stroke should be treated
within the first 3 hours of symptom onset with which of the follow-
ing?
a. Clopidogrel c. Tissue plasminogen activator (tPA)
b. Extended release dipyridamole
c. Tissue plasminogen activator (tPA)
d. Atorvastatin
What is the trade name of Clopidogrel? PLAVIX
What is the drug classification of Clopidogrel and dipyridamole? Antiplatelet- platelet inhibiting meds
What is the trade name for Atorvastatin? LIPITOR
6. A nurse is assisting with a community screening for people at
high risk for stroke. To which of the following clients would the
nurse pay most attention?
a. A 60-year-old African-American man a. A 60-year-old African-American man
b. A 40-year-old Caucasian woman
c. A 62-year-old Caucasian woman
d. A 28-year-old pregnant African-American woman
• Asymptomatic carotid stenosis
• Atrial fibrillation
• Diabetes (associated with accelerated atherogenesis)
• Dyslipidemia
• Excessive alcohol consumption
• Hypercoagulable states
Name three modifiable risk factors for an Ischemic Stroke. • Hypertension (controlling hypertension, the major risk factor, is
the key to preventing stroke)
• Migraine
• Obesity
• Sedentary lifestyle
• Sleep apnea
• Smoking
7. The nurse is reviewing the medication administration record of
a female patient who possesses numerous risk factors for stroke.

16 / 19
STROKE- NCLEX
Study online at https://quizlet.com/_54u2bo
Which of the woman's medications carries the greatest potential
for reducing her risk of stroke?
a. Naproxen 250 PO b.i.d.
c. Aspirin 81 mg PO o.d.
b. Calcium carbonate 1,000 mg PO b.i.d.
c. Aspirin 81 mg PO o.d.
d. Lorazepam 1 mg SL b.i.d. PRN
What is the drug classification for Naproxen? NSAID
What is the drug classification of Aspirin? salicylate___
What is the drug classification of Lorazepam and what is the trade
benzodiazepine, Ativan
name?
8. A nurse in the ICU is providing care for a patient who has
been admitted with a hemorrhagic stroke. The nurse is performing
frequent neurologic assessments and observes that the patient is
becoming progressively more drowsy over the course of the day.
What is the nurse's best response to this assessment finding?
a. Report this finding to the physician as an indication of de-
Report this to the physician as a possible sign of clinical deterio-
creased metabolism.
ration.
b. Provide more stimulation to the patient and monitor the patient
closely.
c. Recognize this as the expected clinical course of a hemorrhagic
stroke.
d. Report this to the physician as a possible sign of clinical
deterioration.
9. An emergency department nurse is interviewing a client with
signs of an ischemic stroke that began 2 hours ago. The client
reports that she had a cholecystectomy 6 weeks ago and is taking
digoxin, coumadin, and labetelol. This client is not eligible for
thrombolytic therapy for which of the following reasons? a. She is taking coumadin.
a. She is taking coumadin.
b. She is not within the treatment time window.
c. She is taking digoxin.
d. She had surgery 6 weeks ago
What do platelets need to be above for Tissue Plasminogen
Platelet count e100,000/mm
Activator (tPA)?
10. From which direction should a nurse approach a client who is
blind in the right eye?
a. From directly in front of the client
c. From the left side of the client
b. From the right side of the client
c. From the left side of the client
d. From directly behind the client
Patients with a decreased field of vision should be approached on
the side where the visual perception is intact or the opposite side Approach the patient from side of intact field of vision.
of the defect?
11. Which of the following is a contraindication for the administra-
tion of tissue plasminogen activator (t-PA)?
a. Intracranial hemorrhage
a. Intracranial hemorrhage
b. Ischemic stroke
c. Age 18 years of age or older
d. Systolic blood pressure less than or equal to 185 mm Hg
12. A client with a history of atrial fibrillation has experienced a
TIA. In an effort to reduce the risk of cerebrovascular accident
(CVA), the nurse anticipates the medical treatment to include
which of the following?
b. Anticoagulant therapy
a. Cholesterol-lowering drugs
b. Anticoagulant therapy
c. Monthly prothrombin levels
d. Carotid endarterectomy

17 / 19
STROKE- NCLEX
Study online at https://quizlet.com/_54u2bo
Name a common medication that patients will be prescribed if
warfarin (Coumadin)
they have atrial fibrillation?
13. A patient is exhibiting classic signs of a hemorrhagic stroke.
What complaint from the patient would be an indicator of this type
of stroke13%
a. Numbness of an arm or leg c. Severe headache
b. Double vision
c. Severe headache
d. Dizziness and tinnitus
14. A 154-pound woman has been prescribed tPA (0.9 mg/kg)
for an ischemic stroke. The nurse knows to give how many mg
initially? a. 6.3 mg - bolus
a. 6.3 mg
b. 7.5 mg 56.7mg- infusion dose
c. 8.3 mg
d. 10 mg
15. What clinical manifestations does the nurse recognize when
a patient has had a right hemispheric stroke?
a. Left visual field deficit
a. Left visual field deficit
b. Aphasia
c. Slow, cautious behavior
d. Altered intellectual ability
16. A physician orders several drugs for a client with hemorrhagic
stroke. Which drug order should the nurse question?
a. Heparin sodium
a. Heparin sodium
b. Dexamethasone (Decadron)
c. Methyldopa (Aldomet)
d. Phenytoin (Dilantin)
Administering heparin, an anticoagulant, could increase the
bleeding associated with hemorrhagic stroke.
TRUE
Therefore, the nurse should question this order (HEPARIN ) to
prevent additional hemorrhage in the brain.
What is Dexamethasone (decadron) used for? to decrease cerebral edema and pressure;
What is methyldopa (Aldomet) used for? to reduce blood pressure
What is phenytoin (Dilantin) used for? to prevent seizures.
17. The nurse is caring for a patient diagnosed with a hemorrhagic
stroke. The nurse recognizes that which of the following interven-
tions is most important?
a. Elevating the head of the bed at 30 degrees d. Maintaining a patent airway
b. Monitoring for seizure activity
c. Administering a stool softener
d. Maintaining a patent airway
TRUE Brain function depends on delivery of oxygen to the tissues
18. The nurse is performing stroke risk screenings at a hospital
open house. The nurse has identified four patients who might be
at risk for a stroke. Which patient is likely at the highest risk for a
hemorrhagic stroke?
b. White male, age 60, with history of uncontrolled hypertension
a. White female, age 60, with history of excessive alcohol intake
b. White male, age 60, with history of uncontrolled hypertension
c. Black male, age 60, with history of diabetes
d. Black male, age 50, with history of smoking
19. A patient who has suffered a stroke begins having compli-
cations regarding spasticity in the lower extremity. What ordered
medication does the nurse administer to help alleviate this prob-
b. Lioresal (Baclofen)
lem?
a. Diphenhydramine (Benadryl)
b. Lioresal (Baclofen)
18 / 19
STROKE- NCLEX
Study online at https://quizlet.com/_54u2bo
c. Heparin
d. Pregabalin (Lyrica)

19 / 19
Stroke NCLEX
Study online at https://quizlet.com/_1d8th5
After a patient experienced a brief episode of tinnitus, diplopia,
C. The patient's symptoms are consistent with transient ischemic
and dysarthria with no residual effects, the nurse anticipates
attack (TIA), and drugs that inhibit platelet aggregation are pre-
teaching the patient about
scribed after a TIA to prevent stroke. Continuous heparin infusion
a. cerebral aneurysm clipping.
is not routinely used after TIA or with acute ischemic stroke. The
b. heparin intravenous infusion.
patient's symptoms are not consistent with a cerebral aneurysm.
c. oral low-dose aspirin therapy.
tPA is used only for acute ischemic stroke, not for TIA.
d. tissue plasminogen activator (tPA).
A 68-year-old patient is being admitted with a possible stroke.
Which information from the assessment indicates that the nurse
should consult with the health care provider before giving the
prescribed aspirin? C. A sudden onset headache is typical of a subarachnoid hemor-
a. The patient has dysphasia. rhage, and aspirin is contraindicated. Atrial fibrillation, dysphasia,
b. The patient has atrial fibrillation. and transient ischemic attack (TIA) are not contraindications to
c. The patient reports that symptoms began with a severe aspirin use, so the nurse can administer the aspirin.
headache.
d. The patient has a history of brief episodes of right-sided hemi-
plegia.
A 73-year-old patient with a stroke experiences facial drooping
on the right side and right-sided arm and leg paralysis. When
admitting the patient, which clinical manifestation will the nurse D. Right-sided paralysis indicates a left-brain stroke, which will
expect to find? lead to difficulty with comprehension and use of language. The
a. Impulsive behavior left-side reflexes are likely to be intact. Impulsive behavior and
b. Right-sided neglect neglect are more likely with a right-side stroke.
c. Hyperactive left-sided tendon reflexes
d. Difficulty comprehending instructions
During the change of shift report a nurse is told that a patient has
an occluded left posterior cerebral artery. The nurse will anticipate
C. Visual disturbances are expected with posterior cerebral artery
that the patient may have
occlusion. Aphasia occurs with middle cerebral artery involve-
a. dysphasia.
ment. Cognitive deficits and changes in judgment are more typical
b. confusion.
of anterior cerebral artery occlusion.
c. visual deficits.
d. poor judgment.
When teaching about clopidogrel (Plavix), the nurse will tell the
D. Clopidogrel (Plavix) inhibits platelet function and increases the
patient with cerebral atherosclerosis
risk for gastrointestinal bleeding, so patients should be advised
a. to monitor and record the blood pressure daily.
to notify the health care provider about any signs of bleeding.
b. that Plavix will dissolve clots in the cerebral arteries.
The medication does not lower blood pressure, decrease plaque
c. that Plavix will reduce cerebral artery plaque formation.
formation, or dissolve clots.
d. to call the health care provider if stools are bloody or tarry.
A patient with carotid atherosclerosis asks the nurse to describe
a carotid endarterectomy. Which response by the nurse is accu-
A. In a carotid endarterectomy, the carotid artery is incised and
rate?
the plaque is removed. The response beginning, "The diseased
a. "The obstructing plaque is surgically removed from an artery in
portion of the artery in the brain is replaced" describes an arterial
the neck."
graft procedure. The answer beginning, "A catheter with a deflated
b. "The diseased portion of the artery in the brain is replaced with
balloon is positioned at the narrow area" describes an angioplasty.
a synthetic graft."
The final response beginning, "A wire is threaded through the
c. "A wire is threaded through an artery in the leg to the clots in
artery" describes the mechanical embolus removal in cerebral
the carotid artery and the clots are removed."
ischemia (MERCI) procedure.
d. "A catheter with a deflated balloon is positioned at the narrow
area, and the balloon is inflated to flatten the plaque."
A patient admitted with possible stroke has been aphasic for 3
D. Because elevated BP may be a protective response to main-
hours and his current blood pressure (BP) is 174/94 mm Hg.
tain cerebral perfusion, antihypertensive therapy is recommended
Which order by the health care provider should the nurse ques-
only if mean arterial pressure (MAP) is >130 mm Hg or systolic
tion?
pressure is >220 mm Hg. Fluid intake should be 1500 to 2000
a. Keep head of bed elevated at least 30 degrees.
mL daily to maintain cerebral blood flow. The head of the bed
b. Infuse normal saline intravenously at 75 mL/hr.
should be elevated to at least 30 degrees, unless the patient has
c. Administer tissue plasminogen activator (tPA) per protocol.
symptoms of poor tissue perfusion. tPA may be administered if the
d. Administer a labetalol (Normodyne) drip to keep BP less than
patient meets the other criteria for tPA use.
140/90 mm Hg.
A 56-year-old patient arrives in the emergency department with
hemiparesis and dysarthria that started 2 hours previously, and
1/5
Stroke NCLEX
Study online at https://quizlet.com/_1d8th5
D. The patient's history and clinical manifestations suggest an
health records show a history of several transient ischemic attacks
acute ischemic stroke and a patient who is seen within 4.5 hours
(TIAs). The nurse anticipates preparing the patient for
of stroke onset is likely to receive tPA (after screening with a
a. surgical endarterectomy.
CT scan). Heparin administration in the emergency phase is
b. transluminal angioplasty.
not indicated. Emergent carotid transluminal angioplasty or en-
c. intravenous heparin administration.
darterectomy is not indicated for the patient who is having an
d. tissue plasminogen activator (tPA) infusion.
acute ischemic stroke.
A female patient who had a stroke 24 hours ago has expressive
aphasia. The nurse identifies the nursing diagnosis of impaired
A. Communication will be facilitated and less frustrating to the
verbal communication. An appropriate nursing intervention to help
patient when questions that require a "yes" or "no" response
the patient communicate is to
are used. When the language areas of the brain are injured,
a. ask questions that the patient can answer with "yes" or "no."
the patient might not be able to read or recite words, which will
b. develop a list of words that the patient can read and practice
frustrate the patient without improving communication. Expressive
reciting.
aphasia is caused by damage to the language areas of the brain,
c. have the patient practice her facial and tongue exercises with a
not by the areas that control the motor aspects of speech. The
mirror.
nurse should allow time for the patient to respond.
d. prevent embarrassing the patient by answering for her if she
does not respond.
A 72-year-old patient who has a history of a transient ischemic
attack (TIA) has an order for aspirin 160 mg daily. When the nurse
is administering medications, the patient says, "I don't need the C. Aspirin is ordered to prevent stroke in patients who have ex-
aspirin today. I don't have a fever." Which action should the nurse perienced TIAs. Documentation of the patient's refusal to take the
take? medication is an inadequate response by the nurse. There is no
a. Document that the aspirin was refused by the patient. need to clarify the order with the health care provider. The aspirin
b. Tell the patient that the aspirin is used to prevent a fever. is not ordered to prevent aches and pains.
c. Explain that the aspirin is ordered to decrease stroke risk.
d. Call the health care provider to clarify the medication order.
For a patient who had a right hemisphere stroke the nurse estab-
A. The patient with right-sided brain damage typically denies any
lishes a nursing diagnosis of
deficits and has poor impulse control, leading to risk for injury
a. risk for injury related to denial of deficits and impulsiveness.
when the patient attempts activities such as transferring from a
b. impaired physical mobility related to right-sided hemiplegia.
bed to a chair. Right-sided brain damage causes left hemiple-
c. impaired verbal communication related to speech-language
gia. Left-sided brain damage typically causes language deficits.
deficits.
Left-sided brain damage is associated with depression and dis-
d. ineffective coping related to depression and distress about
tress about the disability.
disability.
A patient in the clinic reports a recent episode of dysphasia and
B. Following a transient ischemic attack (TIA), patients typically
left-sided weakness at home that resolved after 2 hours. The
are started on medications such as aspirin to inhibit platelet
nurse will anticipate teaching the patient about
function and decrease stroke risk. tPA is used for acute ischemic
a. alteplase (tPA).
stroke. Coumadin is usually used for patients with atrial fibrillation.
b. aspirin (Ecotrin).
Nimodipine is used to prevent cerebral vasospasm after a sub-
c. warfarin (Coumadin).
arachnoid hemorrhage.
d. nimodipine (Nimotop).
When caring for a patient with a new right-sided homonymous
C. During the acute period, the nurse should place objects on the
hemianopsia resulting from a stroke, which intervention should
patient's unaffected side. Because there is a visual defect in the
the nurse include in the plan of care?
right half of each eye, an eye patch is not appropriate. The patient
a. Apply an eye patch to the right eye.
should be approached from the left side. The visual deficit may
b. Approach the patient from the right side.
not resolve, although the patient can learn to compensate for the
c. Place objects needed on the patient's left side.
defect.
d. Teach the patient that the left visual deficit will resolve.
A 58-year-old patient with a left-brain stroke suddenly bursts into
D. Patients who have left-sided brain stroke are prone to emotional
tears when family members visit. The nurse should
outbursts that are not necessarily related to the emotional state
a. use a calm voice to ask the patient to stop the crying behavior.
of the patient. Depression after a stroke is common, but the
b. explain to the family that depression is normal following a stroke.
suddenness of the patient's outburst suggests that depression is
c. have the family members leave the patient alone for a few
not the major cause of the behavior. The family should stay with
minutes.
the patient. The crying is not within the patient's control and asking
d. teach the family that emotional outbursts are common after
the patient to stop will lead to embarrassment.
strokes.
The nurse identifies the nursing diagnosis of imbalanced nutri-
C. Because the nursing diagnosis indicates that the patient's
tion: less than body requirements related to impaired self-feeding
imbalanced nutrition is related to the left-sided hemiplegia, the
ability for a left-handed patient with left-sided hemiplegia. Which
2/5
Stroke NCLEX
Study online at https://quizlet.com/_1d8th5
intervention should be included in the plan of care?
appropriate interventions will focus on teaching the patient to
a. Provide a wide variety of food choices.
use the right hand for self-feeding. The other interventions are
b. Provide oral care before and after meals.
appropriate for patients with other etiologies for the imbalanced
c. Assist the patient to eat with the right hand.
nutrition.
d. Teach the patient the "chin-tuck" technique.
Which stroke risk factor for a 48-year-old male patient in the clinic
C. Hypertension is the single most important modifiable risk factor.
is most important for the nurse to address?
People who drink more than 1 (for women) or 2 (for men) alcoholic
a. The patient is 25 pounds above the ideal weight.
beverages a day may increase risk for hypertension. Physical
b. The patient drinks a glass of red wine with dinner daily.
inactivity and obesity contribute to stroke risk but not as much as
c. The patient's usual blood pressure (BP) is 170/94 mm Hg.
hypertension.
d. The patient works at a desk and relaxes by watching television.
A 40-year-old patient has a ruptured cerebral aneurysm and A. The patient with a subarachnoid hemorrhage usually has mini-
subarachnoid hemorrhage. Which intervention will be included in mal activity to prevent cerebral vasospasm or further bleeding and
the care plan? is at risk for venous thromboembolism (VTE). Activities such as
a. Apply intermittent pneumatic compression stockings. coughing and sitting up that might increase intracranial pressure
b. Assist to dangle on edge of bed and assess for dizziness. (ICP) or decrease cerebral blood flow are avoided. Because there
c. Encourage patient to cough and deep breathe every 4 hours. is no indication that the patient is unconscious, an oropharyngeal
d. Insert an oropharyngeal airway to prevent airway obstruction. airway is inappropriate.
A patient in the emergency department with sudden-onset D. The use of warfarin probably contributed to the intracerebral
right-sided weakness is diagnosed with an intracerebral hemor- bleeding and remains a risk factor for further bleeding. Admin-
rhage. Which information about the patient is most important to istration of vitamin K is needed to reverse the effects of the
communicate to the health care provider? warfarin, especially if the patient is to have surgery to correct
a. The patient's speech is difficult to understand. the bleeding. The history of hypertension is a risk factor for the
b. The patient's blood pressure is 144/90 mm Hg. patient but has no immediate effect on the patient's care. The BP
c. The patient takes a diuretic because of a history of hyperten- of 144/90 indicates the need for ongoing monitoring but not for any
sion. immediate change in therapy. Slurred speech is consistent with a
d. The patient has atrial fibrillation and takes warfarin (Coumadin). left-sided stroke, and no change in therapy is indicated.
A 47-year-old patient will attempt oral feedings for the first time
C. The patient should be as upright as possible before attempting
since having a stroke. The nurse should assess the gag reflex and
feeding to make swallowing easier and decrease aspiration risk.
then
To assess swallowing ability, the nurse should initially offer water
a. order a varied pureed diet.
or ice to the patient. Pureed diets are not recommended because
b. assess the patient's appetite.
the texture is too smooth. The patient may have a poor appetite,
c. assist the patient into a chair.
but the oral feeding should be attempted regardless.
d. offer the patient a sip of juice.
A patient with left-sided weakness that started 60 minutes earlier D. Rapid screening with a noncontrast CT scan is needed before
is admitted to the emergency department and diagnostic tests are administration of tissue plasminogen activator (tPA), which must
ordered. Which test should be done first? be given within 4.5 hours of the onset of clinical manifestations of
a. Complete blood count (CBC) the stroke. The sooner the tPA is given, the less brain injury. The
b. Chest radiograph (Chest x-ray) other diagnostic tests give information about possible causes of
c. 12-Lead electrocardiogram (ECG) the stroke and do not need to be completed as urgently as the CT
d. Noncontrast computed tomography (CT) scan scan.
A male patient who has right-sided weakness after a stroke is
making progress in learning to use the left hand for feeding and
other activities. The nurse observes that when the patient's wife
is visiting, she feeds and dresses him. Which nursing diagnosis is C. The information supports the diagnosis of disabled family
most appropriate for the patient? coping because the wife does not understand the rehabilitation
a. Interrupted family processes related to effects of illness of a program. There are no data supporting low self-esteem, and the
family member patient is attempting independence. The data do not support an
b. Situational low self-esteem related to increasing dependence interruption in family processes because this may be a typical
on spouse for care pattern for the couple. There is no indication that the patient has
c. Disabled family coping related to inadequate understanding by impaired nutrition.
patient's spouse
d. Impaired nutrition: less than body requirements related to hemi-
plegia and aphasia
Nurses in change-of-shift report are discussing the care of a
patient with a stroke who has progressively increasing weakness D. Protection of the airway is the priority of nursing care for a
and decreasing level of consciousness (LOC). Which nursing di- patient having an acute stroke. The other diagnoses are also
agnosis do they determine has the highest priority for the patient? appropriate, but interventions to prevent aspiration are the priority
a. Impaired physical mobility related to weakness at this time.
b. Disturbed sensory perception related to brain injury
3/5
Stroke NCLEX
Study online at https://quizlet.com/_1d8th5
c. Risk for impaired skin integrity related to immobility
d. Risk for aspiration related to inability to protect airway
Several weeks after a stroke, a 50-year-old male patient has
impaired awareness of bladder fullness, resulting in urinary in-
B. Developing a regular voiding schedule will prevent inconti-
continence. Which nursing intervention will be best to include in
nence and may increase patient awareness of a full bladder. A
the initial plan for an effective bladder training program?
1200 mL fluid restriction may lead to dehydration. Intermittent
a. Limit fluid intake to 1200 mL daily to reduce urine volume.
catheterization and use of a condom catheter are appropriate in
b. Assist the patient onto the bedside commode every 2 hours.
the acute phase of stroke, but should not be considered solutions
c. Perform intermittent catheterization after each voiding to check
for long-term management because of the risks for urinary tract
for residual urine.
infection (UTI) and skin breakdown.
d. Use an external "condom" catheter to protect the skin and
prevent embarrassment.
Which information about the patient who has had a subarachnoid B. To prevent cerebral vasospasm and maintain cerebral perfu-
hemorrhage is most important to communicate to the health care sion, blood pressure needs to be maintained at a level higher
provider? than 90 mm Hg systolic after a subarachnoid hemorrhage. A low
a. The patient complains of having a stiff neck. BP or drop in BP indicates a need to administer fluids and/or
b. The patient's blood pressure (BP) is 90/50 mm Hg. vasopressors to increase the BP. An ongoing headache, RBCs
c. The patient reports a severe and unrelenting headache. in the CSF, and a stiff neck are all typical clinical manifestations
d. The cerebrospinal fluid (CSF) report shows red blood cells of a subarachnoid hemorrhage and do not need to be rapidly
(RBCs). communicated to the health care provider.
The nurse is caring for a patient who has been experiencing stroke
symptoms for 60 minutes. Which action can the nurse delegate to
C. Administration of subcutaneous medications is included in
a licensed practical/vocational nurse (LPN/LVN)?
LPN/LVN education and scope of practice. The other actions
a. Assess the patient's gag and cough reflexes.
require more education and scope of practice and should be done
b. Determine when the stroke symptoms began.
by the registered nurse (RN).
c. Administer the prescribed short-acting insulin.
d. Infuse the prescribed IV metoprolol (Lopressor).
After receiving change-of-shift report on the following four pa-
tients, which patient should the nurse see first?
a. A 60-year-old patient with right-sided weakness who has an
infusion of tPA prescribed A. tPA needs to be infused within the first few hours after stroke
b. A 50-year-old patient who has atrial fibrillation and a new order symptoms start in order to be effective in minimizing brain injury.
for warfarin (Coumadin) The other medications should also be given as quickly as possible,
c. A 40-year-old patient who experienced a transient ischemic but timing of the medications is not as critical.
attack yesterday who has a dose of aspirin due
d. A 30-year-old patient with a subarachnoid hemorrhage 2 days
ago who has nimodipine (Nimotop) scheduled
The nurse is caring for a patient who has just returned after having B. Small emboli can occur during carotid artery angioplasty and
left carotid artery angioplasty and stenting. Which assessment stenting, and the aphasia indicates a possible stroke during the
information is of most concern to the nurse? procedure. Slightly elevated pulse rate and blood pressure are
a. The pulse rate is 102 beats/min. not unusual because of anxiety associated with the procedure.
b. The patient has difficulty speaking. Fine crackles at the lung bases may indicate atelectasis caused
c. The blood pressure is 144/86 mm Hg. by immobility during the procedure. The nurse should have the
d. There are fine crackles at the lung bases. patient take some deep breaths.
A 70-year-old female patient with left-sided hemiparesis arrives
by ambulance to the emergency department. Which action should
C. The initial nursing action should be to assess the airway and
the nurse take first?
take any needed actions to ensure a patent airway. The other ac-
a. Monitor the blood pressure.
tivities should take place quickly after the ABCs (airway, breathing,
b. Send the patient for a computed tomography (CT) scan.
and circulation) are completed.
c. Check the respiratory rate and effort.
d. Assess the Glasgow Coma Scale score.
The home health nurse is caring for an 81-year-old who had a
B. The spouse's household and patient care responsibilities, in
stroke 2 months ago. Based on information shown in the ac-
combination with chronic illnesses, indicate a high risk for care-
companying figure from the history, physical assessment, and
giver role strain. The nurse should further assess the situation
physical therapy/occupational therapy, which nursing diagnosis is
and take appropriate actions. The data about the control of the
the highest priority for this patient?
patient's diabetes indicates that ineffective health maintenance
a. Impaired transfer ability
and risk for unstable blood glucose are not priority concerns at
b. Risk for caregiver role strain
this time. Because the patient is able to ambulate with a cane, the
c. Ineffective health maintenance
nursing diagnosis of impaired transfer ability is not supported.
d. Risk for unstable blood glucose level

4/5
Stroke NCLEX
Study online at https://quizlet.com/_1d8th5
A 63-year-old patient who began experiencing right arm and leg
weakness is admitted to the emergency department. In which
C, D, A, B
order will the nurse implement these actions included in the stroke
The initial actions should be those that help with airway, breathing,
protocol?
and circulation. Baseline neurologic assessments should be done
a. Obtain computed tomography (CT) scan without contrast.
next. A CT scan will be needed to rule out hemorrhagic stroke
b. Infuse tissue plasminogen activator (tPA).
before tPA can be administered.
c. Administer oxygen to keep O2 saturation >95%.
d. Use National Institute of Health Stroke Scale to assess patient.

5/5

You might also like